Вы находитесь на странице: 1из 52

PSIQUIATRA-3

AVALADO POR

PROGRAMA DE ACTUALIZACIN CONTINUA EN PSIQUIATRA

COMIT EJECUTIVO 2000 - 2001

PRESIDENTE
Dr. Enrique Camarena Robles
PRESIDENTE ELECTO
Dr. Marco Antonio Lpez Butrn
SECRETARIO GENERAL
Dr. Luis Enrique Rivero Almanzar
TESORERO
Dr. Eduardo Nez Bernal
VICEPRESIDENCIA NORTE

DIFERENCIAS DE GNERO
Dr. Hctor Gonzlez Vargas
VICEPRESIDENCIA CENTRO
Dr. Juan Luis Vzquez Hernndez
VICEPRESIDENCIA SUR
Dr. Arsenio Rosendo Franco
SECRETARIO DE ASUNTOS INTERNOS
Dr. Jos Daz Martnez
SECRETARIO AUXILIAR
Hctor Rodrguez Jurez
Y CEREBRO
COORDINACIN INTERESTATAL
COORDINADOR
Dr. Oscar Benassini Flix
SECRETARIO AUXILIAR
Dr. Jos Luis Esquinca Ramos
ACTIVIDADES CIENTFICAS NACIONALES
COORDINADORA
Dra. Rosa Isela Mzquita
SECRETARIO AUXILIAR
Dr. Miguel Herrera Estrella
ACTIVIDADES CIENTFICAS INTERNACIONALES
COORDINADOR

LIBRO 1
Dr. Enrique Chvez Len
SECRETARIOS AUXILIARES
Dra. Ma. del Carmen Lara Muoz
Dra. Martha Ontiveros Uribe
Dr. Oscar Snchez Guerrero
COORDINACIN DE MEMBRESA Y ASUNTOS GREMIALES
COORDINADOR
Dr. Javier Alfaro Torres
SECRETARIOS AUXILIARES
Dr. Juan Carlos Rueda
Dr. Jess Alba Nieto
COORDINACIN OPERATIVA DE EVENTOS ACADMICOS
COORDINADOR
Dr. Alejandro Crdova Castaeda
SECRETARIOS AUXILIARES
Dr. Rogelio Gallegos Casares
Dr. Carlos Lpez Elizalde
AUTORES
COORDINACIN INTERINSTITUCIONAL
COORDINADOR
DRA. MARTHA ONTIVEROS URIBE
Dr. Jess del Bosque Garza
SECRETARIA AUXILIAR
DRA. MARA DEL CARMEN LARA MUOZ
Alejandra Vzquez Ramrez
COORDINACIN DE EDUCACIN CONTINUA
COORDINADOR
Dr. Francisco Lorenzo Martnez
SECRETARIO AUXILIAR
Dr. Fernando Lpez Mungua
COORDINACIN DE EVALUACIN Y DESARROLLO
DE INVESTIGACIONES
COORDINADOR
Dr. Leopoldo Zrate Hidalgo
SECRETARIO AUXILIAR
Dr. Fernando Corona Hernndez
COORDINACIN DE PUBLICACIONES
COORDINADOR
Dr. Marco Antonio Dupont
COORDINACIN DE SECCIONES PERMANENTES
Y CAPTULOS ESTATALES
COORDINADOR
Dr. Juan Pablo Fernndez Hernndez
UN PROGRAMA EDUCATIVO
SECRETARIO AUXILIAR
Dr. Carlos Gutirrez de Aquino PATROCINADO POR
EDITOR DE LA REVISTA PSIQUIATRA
Dr. Hctor Ortega Soto
AUTORES
DRA. MARTHA ONTIVEROS URIBE
Psiquiatra UNAM
Subdirectora del Servicio Hospitalario del Instituto Nacional de Psiquiatra
Catedrtica de la UNAM
Catedrtica de la Escuela de Psicologa de la Universidad Anhuac
Coordinadora de la seccin permanente de "Mujer Psiquiatra" de la Asociacin Psiquitrica Mexicana A.C.

DRA. MARA DEL CARMEN LARA MUOZ


Psiquiatra UAP
Presidenta del Consejo Mexicano de Psiquiatra
Vocal de la seccin permanente de "Mujer Psiquiatra" de la Asociacin Psiquitrica Mexicana A.C.

UNA EDICIN DE
INTERSISTEMAS, S.A. DE C.V.

EDUCACIN MDICA CONTINUA

PAC PSIQUIATRA-3
Primera Edicin 2002
Copyright 2002 Intersistemas, S.A. de C.V.
Todos los derechos reservados. Este libro est protegido por los derechos de autor. Ninguna parte de esta publicacin puede ser reproducida, almacenada en algn sistema de recuperacin, o transmi-
tida de ninguna forma o por ningn medio, electrnico o mecnico, incluyendo fotocopia, sin autorizacin previa del editor.
PAC Es una marca registrada de Intersistemas S. A. de C. V.

ISBN 970-655-342-8 Edicin completa


ISBN 970-655-343-6 Libro 1
Impreso en Mxico
El contenido del programa PAC PSIQUIATRIA-3 es responsabilidad exclusiva de los autores. El editor no se responsabiliza de ninguno de los conceptos, recomendaciones, dosis, etc. vertidos por los
autores y su aplicacin queda a criterio de los lectores.
Diseo de portada: Edgar Romero Escobar
Diagramacin: Dulce Mara Lomel
PRESENTACIN

ESTIMADOS COLEGAS, SOCIOS Y PROFESIONALES DE LA SALUD MENTAL:

E
n los ltimos decenios hemos aprendido que la SALUD MENTAL representa una de las l-
timas fronteras que es preciso franquear para mejorar la calidad de vida, elemento que ma-
tiza y da valor a la condicin humana. En paralelo, hemos logrado redimensionar a los tras-
tornos mentales gracias a los estudios epidemiolgicos, los avances en la capacidad diagnstica de
los clnicos y la ponderacin de los costos econmicos y sociales generados por los trastornos men-
tales. Mundialmente, en los pases desarrollados y subdesarrollados estos trastornos considerados
nocivos para la salud individual y colectiva, son causa de sufrimiento y desintegracin social y fa-
miliar. ste se manifiesta en la desesperanza y afliccin de los individuos, la angustia de las fami-
lias y las prdidas econmicas y sociales dadas por la disminucin de la actividad productiva que
infligen sobre los individuos las enfermedades mentales y por el aumento en el uso de los servicios
de salud y de asistencia social en los diferentes pases.
Para vencer estos problemas, los retos son grandes. Las enfermedades mentales tienen sus cau-
sas y orgenes en mltiples variables. Su centro de estudio radica en el cerebro, rgano de alta com-
plejidad tanto por su organizacin biolgica intrincada, como por sus sutiles relaciones con otros
rganos del sistema corporal y el medio ambiente con el que interacta. En algunos casos hemos
hecho avances importantes. Sin embargo, las causas finales de enfermedades tan dramticas como
la esquizofrenia y el autismo todava siguen siendo un misterio. Con esperanza hemos sido testigos
en los ltimos aos de adelantos en materia de neurobioqumica y neurofisiologa, as como en las
tcnicas de intervencin psicolgica, lo que ha derivado en tratamientos ms eficaces en psicofar-
macoterapia y psicoterapia. El arsenal teraputico se ha multiplicado y los conocimientos ms es-
pecficos y detallados sobre los mecanismos moleculares de las enfermedades mentales, desentraan
dudas e iluminan el sendero para dar paso a nuevos descubrimientos que seguramente sern la ba-
se para el desarrollo de nuevos modelos de prevencin, tratamiento y rehabilitacin.
Hemos empezado a identificar cules son las tcnicas psicoteraputicas ms efectivas para de-
terminadas enfermedades. Al medir con ms exactitud el impacto de nuestras acciones, hemos lo-
grado jerarquizar las tcnicas de intervencin psicolgica, abandonando poco a poco el empirismo
que ha sido motivo de cuestionamientos y de dudas. La terapia breve y de urgencia y la terapia cog-
nitivo conductual, son ejemplos de tcnicas tiles para un sector amplio de la poblacin con nece-
sidades emocionales especficas que requieren de una ayuda inmediata, derivadas de enfermedades
que les hacen sufrir e incapacita. Las teoras psicodinmicas han evolucionado, se han enriquecido
con la experiencia de los aos, y con sabidura han afinado sus procesos. Hemos aprendido a dar
valor real a nuestras diferentes disciplinas y esto ha derivado en la creacin de los equipos interdis-
ciplinarios y multidisciplinarios.
Por lo tanto es imperioso seguir en un proceso de educacin continua, en donde los avances en
las diversas disciplinas relacionadas con la psiquiatra y la prevencin y promocin de la salud men-
tal, estn a la mano del profesional del rea, en nuestro idioma general y con un enfoque didctico.
En esta nueva serie PAC, la Asociacin Psiquitrica Mexicana ha hecho un esfuerzo para con-
tinuar con esta tarea; esperamos que sus expectativas sean satisfechas. Debe mencionarse la partici-
pacin de Pfizer-Sistema Nervioso Central e Intersistemas Editores para el logro de esta publicacin.

Dr. Enrique Camarena Robles


Presidente de APM
CONTENIDO
AUTOEVALUACIN INICIAL . . . . . . . . . . . . . . . . . . . . . . . . . 7
DIFERENCIAS DE GNERO Y CEREBRO . . . . . . . . . . . . . . . . . . . 9
PREVALENCIA . . . . . . . . . . . . . . . . . . . . . . . . . . . . . . . . 20
HIPTESIS QUE TRATAN DE EXPLICAR LA ASOCIACIN
ENTRE LOS ESTRGENOS Y LOS TRASTORNOS PSIQUITRICOS . 22
UTILIZACIN DE LOS SERVICIOS DE SALUD . . . . . . . . . . . . 22
SESGO EN LAS RESPUESTAS DADAS EN LAS ENTREVISTAS . . . 23
SESGO EN EL DIAGNSTICO Y EN EL RECUERDO . . . . . . . . 23
EQUIVALENTES DEPRESIVOS . . . . . . . . . . . . . . . . . . . . . 24
GENTICAS . . . . . . . . . . . . . . . . . . . . . . . . . . . . . . . 24
ENDOCRINAS-HORMONALES . . . . . . . . . . . . . . . . . . . . . 24
ESTATUS SOCIAL . . . . . . . . . . . . . . . . . . . . . . . . . . . . 30
EXPOSICIN Y VULNERABILIDAD A EVENTOS
ADVERSOS DE LA VIDA ...................... 31
APOYO SOCIAL . . . . . . . . . . . . . . . . . . . . . . . . . . . . . 31
COGNOSCITIVOS. DESESPERANZA APRENDIDA . . . . . . . . . . 31
COGNOSCITIVOS. ESTILOS DE RESPUESTA . . . . . . . . . . . . . 32
DIFERENCIAS DE GNERO EN OTROS
TRASTORNOS PSIQUITRICOS . . . . . . . . . . . . . . . . . . . . . . 32
TRASTORNO BIPOLAR . . . . . . . . . . . . . . . . . . . . . . . . . 32
ESQUIZOFRENIA . . . . . . . . . . . . . . . . . . . . . . . . . . . . 33
TRASTORNO DE ANSIEDAD . . . . . . . . . . . . . . . . . . . . . . 34
ALCOHOLISMO Y ABUSO DE SUSTANCIAS . . . . . . . . . . . . . 35
TRASTORNOS DE LA PERSONALIDAD . . . . . . . . . . . . . . . . 36
DESARROLLO DE LA IDENTIDAD DE GNERO . . . . . . . . . . . . 36
INFLUENCIA DE LOS PADRES . . . . . . . . . . . . . . . . . . . . . 37
DESARROLLO COGNOSCITIVO DEL CEREBRO . . . . . . . . . . . 38
ESTEREOTIPOS GENRICOS . . . . . . . . . . . . . . . . . . . . . . . . 39
NEURASTENIA . . . . . . . . . . . . . . . . . . . . . . . . . . . . . . . 41
ORIENTACIN SEXUAL . . . . . . . . . . . . . . . . . . . . . . . . . . 43
BIBLIOGRAFA . . . . . . . . . . . . . . . . . . . . . . . . . . . . . . . . . 47
AUTOEVALUACIN FINAL . . . . . . . . . . . . . . . . . . . . . . . . . . 49
RESPUESTAS DE AUTOEVALUACIN INICIAL Y FINAL . . . . . . . . . . 51
AUTOEVALUACIN INICIAL

1 En los seres humanos el dimorfismo sexual se manifiesta cl- Ver respuestas


nicamente a nivel genital. en la pgina 51
Falso Verdadero

2 Los ncleos supraquiasmtico e intersticial del hipotlamo


son de mayor tamao en los hombres que en las mujeres.
Falso Verdadero

3 Las mujeres tienen una menor fluidez verbal que los hombres.
Falso Verdadero

4 La parte posterior del cuerpo calloso es mayor en las mujeres


que en los hombres.
Falso Verdadero

5 El riesgo de recurrencia para desarrollar depresin en el pos-


parto es de 50%.
Falso Verdadero

6 En los hombres la depresin tiende a preceder al alcoholismo


y en las mujeres tiende a ser una consecuencia.
Falso Verdadero

7 Son mujeres 75% de las personas con trastorno lmite de la


personalidad.
Falso Verdadero

8 Los hombres tienen mayor habilidad en la rotacin espacial y


en las matemticas que las mujeres.
Falso Verdadero

9 Se ha propuesto que el orden de las etapas del ciclo vital pro-


puestas por Erickson es diferente para hombres y mujeres. En
las mujeres la formacin de la identidad precede a la etapa de in-
timidad y para los hombres la intimidad precede a la identidad.
Falso Verdadero

10 El padre est ms involucrado en la socializacin de los hijos


que de las hijas.
Falso Verdadero 7
DIFERENCIAS DE GNERO
Y CEREBRO

E
n aos recientes, el inters en las Una dificultad adicional para estu-
El conocimiento de las
diferencias entre los sexos en la diar las diferencias de gnero en el cere- diferencias en la morfologa
morfologa cerebral se ha incre- bro humano, es tomar en cuenta las in- cerebral entre hombres y
mentado, concomitantemente a la r- fluencias del medio ambiente. El mujeres, es crucial para controlar
pida evolucin de la tecnologa de las cerebro es completamente plstico y los efectos del sexo, cuando se
imgenes cerebrales. Las imgenes es- responde tanto estructural como fun- estudia la estructura y la funcin
tructurales se obtienen por mtodos cionalmente a las influencias medio cerebrales afectadas en un
como la tomografa axial computariza- ambientales. Desde el momento del trastorno en particular.
da (TAC) o la resonancia magntica nacimiento, el gnero de asignacin
nuclear (RMN). Las imgenes funcio- condiciona formas de trato y expectati-
nales se obtienen por mtodos que eva- vas, de los padres, de la sociedad y de la
lan el flujo sanguneo o el metabolis- cultura, diferentes para los nios y las
mo cerebral, como indicadores nias. An as, estudios en recin naci-
especficos de la actividad cerebral, por dos, muestran diferencias en ambos se-
ejemplo la tomografa computada por xos en: la respuesta sensorial; la fuerza,
emisin de fotn nico (SPECT), la el patrn y la simetra de los movimien-
tomografa por emisin de positrones tos; vocalizacin, agudeza visual y tiem-
(PET), y la resonancia magntica nu- po de contacto de la mirada. Estas dife-
clear funcional (fRMN). rencias tan tempranas en los recin
Las razones del inters en las dife- nacidos, en quienes la influencia medio
rencias entre los sexos en el cerebro son ambiental y social es mnima, sugieren
las siguientes: la existencia de un sustrato biolgico en
1. Entender cmo difieren la estruc- las diferencias de gnero cerebrales y su
tura y la funcin cerebrales entre relacin con la conducta.
hombres y mujeres. Los efectos del gnero sobre el ce-
2. El conocimiento de las diferencias en rebro se deben estudiar desde una pers-
la morfologa cerebral entre hombres pectiva del desarrollo a lo largo del ci-
y mujeres, es crucial para controlar los clo vital, desde la gestacin hasta la
efectos del sexo, cuando se estudia la vejez, sobre todo porque sabemos que
estructura y la funcin cerebrales afec- el crecimiento y la maduracin del ce- Hasta las seis semanas de
tadas en un trastorno en particular. rebro humano son procesos que conti- gestacin, los embriones
Por ejemplo las alteraciones cerebrales nan a lo largo de muchos aos des- masculinos y femeninos, tienen
identificadas en la esquizofrenia, las pus del nacimiento, en la adolescencia gnadas indiferenciadas y son
cuales tienden a ser ms marcadas en y an en la vida adulta. morfolgicamente idnticos.
hombres que en mujeres. Hasta las seis semanas de gesta-
3. La necesidad de entender cmo cin, los embriones masculinos y fe-
las diferencias entre los sexos inte- meninos, tienen gnadas indiferencia-
ractan con los diferentes trastor- das y son morfolgicamente idnticos.
nos psiquitricos, para provocar A partir de entonces, influyen en la de-
las diferencias de gnero en la fe- terminacin del sexo y el gnero las si-
nomenologa de dichos trastor- guientes caractersticas:
nos: presentacin, curso, severi- 1. Genticas. De acuerdo al patrn de
dad, respuesta a tratamiento. cromatina sexual. El sujeto es mas-

9
PAC PSIQUIATRA-3 LIBRO 1

culino si tiene un par de cromoso- en el hombre un miembro del par es


mas sexuales XY y es femenino si largo X y el otro corto Y.
tiene un par de cromosomas sexua- El sexo est determinado por un
les XX. gen situado en el cromosoma Y; este
2. Gonadales. De acuerdo al tejido gen se conoce como factor determi-
gonadal presente. En el sexo mas- nante de los testculos (TDF por sus si-
culino las gnadas son los testcu- glas en ingls testis determining factor).
El sexo est determinado por un los. Y en el sexo femenino las gna- Si el individuo tiene un cromosoma Y,
gen situado en el cromosoma Y; das son los ovarios. tiene el gen TDF, entonces se desarro-
este gen se conoce como factor 3. Hormonales. De acuerdo a las hor- lla como hombre. Si no tiene el cro-
determinante de los testculos monas presentes. Las hormonas mosoma Y, por lo tanto no tiene el gen
(TDF por sus siglas en ingls consideradas como masculinas son TDF, y se desarrolla como mujer.
testis determining factor). los andrgenos: testosterona y di- El gen TDF trabaja sobre un solo
hidrotestosterona. Las hormonas tejido, las gnadas masculinas o testcu-
consideradas como femeninas son los. Si dicho gen no est presente, las
los estrgenos y la progesterona. gnadas se desarrollan como femeninas
4. La morfologa de los genitales in- u ovarios. Durante las primeras semanas
ternos. En el sexo masculino estn de la gestacin, las gnadas son indife-
presentes la prstata, las vesculas renciadas y estn cerca de los riones en
seminales y el conducto deferente. la parte posterior de la cavidad abdomi-
En el sexo femenino los ovarios, las nal. Al cumplirse seis semanas de gesta-
trompas de Falopio y el tero. cin, si el TDF est ausente las gnadas
5. La morfologa de los genitales exter- se desarrollan como ovarios y si est pre-
nos. En el sexo masculino estn pre- sente se desarrollan como testculos.
sentes los testculos y el epiddimo Si el TDF est ausente la trayecto-
contenidos en el escroto y el pene. ria de desarrollo es femenina. Las clu-
En el sexo femenino la vagina, los la- las alrededor de los ovarios se convier-
bios mayores y menores y el cltoris. ten en clulas tecales que ms tarde
6. El sexo de asignacin y de crianza. sintetizarn hormonas esteroideas fe-
La forma en la que el individuo es meninas o estrgenos. Las estructuras
educado de acuerdo al sexo asignado embrionarias llamadas conductos de
por sus genitales externos. Por lo ge- Mller se transformarn en las trom-
neral a los nios se les permiten los pas de Falopio, el tero, el crvix y los
juegos ms activos: saltar, brincar, el genitales externos.
contacto fsico durante el juego con Si el TDF est presente, acta sobre
otros nios. Y las nias juegan con las clulas de Sertoli, que a su vez envan
muecas, a la "comidita", y se les re- un mensaje a las clulas cercanas, deno-
comienda no ensuciar su ropa. minadas de Leydig, para que produzcan
7. El sexo psicolgico o papel genri- hormonas esteroideas masculinas, an-
co. El rol que cada individuo adop- drgenos testosterona. Entre las sema-
ta segn se asume como hombre o nas nueve y la dieciocho de gestacin,
como mujer. los andrgenos secretados por los test-
Las caractersticas mencionadas culos diferencian al feto como masculi-
van interactuando a lo largo de la vida, no. La testosterona acta directamente
como se describe a continuacin: sobre los conductos de Wolf para que se
El ncleo de las clulas humanas desarrollen los genitales internos mascu-
contiene 23 pares de cromosomas, es linos, e indirectamente a travs de la di-
decir, tiene un total de 46 cromoso- hidrotestosterona para que se desarro-
mas. De esos 23 pares, 22 son autoso- llen el escroto y el pene. Las clulas de
mas y un par sexual: en la mujer XX, y Sertoli secretan la hormona inhibidora

10
Diferencias de gnero y cerebro

de los conductos mllerianos. La testos- Estos individuos desarrollan un


terona es detectada por primera vez en patrn cognoscitivo "tpicamente fe-
la sangre del feto humano masculino a menino", con un mayor desarrollo
los dos meses de gestacin. del lenguaje y un menor desarrollo
Una vez que se ha determinado el de la orientacin espacial. Esto su-
sexo de las gnadas, procede la diferen- giere que la falta de andrgenos hace
ciacin sexual del resto del cuerpo. Es- que el cerebro se diferencie como fe-
ta diferenciacin est determinada di- menino, con el patrn cognoscitivo
rectamente por las hormonas secretadas correspondiente.
a partir de las gnadas y slo indirecta- 2. Hiperplasia adrenal congnita.
mente por los genes. Por ejemplo, el ge- Es una anormalidad congnita, en
notipo de un embrin puede ser mas- la que las glndulas adrenales secre-
culino y tendr testculos; pero si los tan una gran cantidad de andrge-
testculos no producen testosterona o s nos, que masculinizan al feto feme-
sta no es efectiva en su accin sobre los nino. Este exceso de andrgenos no
tejidos, el feto se desarrollar con un fe- afecta los genitales internos, pero si
notipo femenino, aunque el genotipo los externos, que se masculinizan
sea masculino. Si un feto de genotipo en grado variable. Y tambin se
femenino es expuesto al efecto de la tes- masculiniza parcialmente el cere-
tosterona en un perodo crtico, desa- bro. Las nias con esta alteracin
rrollar un fenotipo masculino. son ms traviesas, agresivas, y "reto-
Las anormalidades que se presentan zonas", que sus hermanas. Tienen Las anormalidades que se
en el desarrollo sexual del feto, generan predileccin por los juguetes pro- presentan en el desarrollo sexual
alteraciones denominadas estados inter- pios de nios. Desarrollan ms las del feto, generan alteraciones
sexuales, caracterizados por ambigedad habilidades viso-espaciales de ma- denominadas estados
de los genitales internos, de los genitales nipulacin y rotacin, habilidades intersexuales, caracterizados por
externos, y por conductas peculiares en que habitualmente desarrollan me- ambigedad de los genitales
los individuos que las padecen. jor los nios. Tienden ms a ser internos, de los genitales
Ejemplos de alteraciones o estados zurdas. Y tienen mayor probabili- externos, y por conductas
peculiares en los individuos que
intersexuales: dad de ser homo o bisexuales, que
las padecen.
1. Sndrome del testculo femini- las nias no expuestas a andrge-
zante. Si la hormona inhibidora de nos. Al compararlas con otras ni-
los conductos mllerianos (MIH) as, no presentan diferencias verba-
est ausente o no es efectiva, pero les, ni de razonamiento.
hay testosterona o dihidrotestoste- 3. Deficiencia en la enzima 5 alfa
rona, el embrin desarrolla genita- reductasa. Esta enzima convierte
les internos masculinos, y genitales la testosterona en dihidrotestoste-
externos femeninos. Es decir el in- rona, la cual se une al receptor de
dividuo tiene un genotipo masculi- andrgenos ms efectivamente
no, pero un fenotipo femenino. que la testosterona. Si la dihidros-
Por lo tanto el nio es criado como testosterona no est presente, los
nia, y no es hasta la pubertad genitales internos se desarrollan
cuando no menstra, que se le es- como masculinos, pero los genita-
tudia, y se identifica que de acuer- les externos se masculinizan en
do a la informacin cromosmica forma insuficiente. El individuo
y a los genitales internos, es un va- parece mujer, o sus genitales ex-
rn y no una mujer. El mdico en- ternos tienen un aspecto interme-
frenta un conflicto al tener que in- dio entre masculinos y femeninos.
formar dicho hallazgo al paciente y Al llegar el sujeto a la pubertad,
a la familia. cuando aumenta el nivel de an-

11
PAC PSIQUIATRA-3 LIBRO 1

drgenos, el tejido genital externo Los testculos se desarrollan en el


responde a stos, y pareciera que feto masculino en la sexta semana de la
el sexo del individuo cambia de gestacin e inician la produccin de
femenino a masculino, con el andrgenos; a partir de ese momento
consecuente impacto psicolgico el cerebro masculino y el femenino se
de haberse desarrollado como mu- empiezan a diferenciar.
jer y realmente ser hombre. La sensibilidad mxima del SNC al
Aunque algunos estudios sugieren efecto organizacional temprano de los
que la exposicin a las hormonas se- esteroides gonadales ocurre presumi-
xuales, conforma conductas que se blemente entre las semanas 14 y 16 de
manifestarn a lo largo de la vida, hay la gestacin, cuando est presente el pi-
muy poca evidencia en estudios en se- co en la concentracin de testosterona.
res humanos que permita conclusio- La testosterona entra al cerebro mas-
nes contundentes. culino y se une al receptor hormonal, de-
Los ovarios en la vida fetal no se- sencadenando una cascada de efectos
cretan niveles significativos de estrge- que modelarn de manera permanente el
nos; en ausencia de esteroides el cuer- cerebro masculino, confirindole carac-
po se desarrolla en direccin femenina. tersticas de algn modo distintas al cere-
Se puede decir que el desarrollo feme- bro femenino, en la estructura, en la fun-
nino se da en ausencia de instrucciones cin, y en el tiempo del neurodesarrollo.
especficas de lo contrario. En la pu- La testosterona ocupa directamente,
bertad los niveles de estrgenos son al- o como dihidrotestosterona, receptores
tos y el cuerpo termina de desarrollar- cerebrales intranucleares. Se trata de un
se como femenino. En el hombre el complejo receptor-hormona-DNA, que
aumento de los niveles de testosterona da las instrucciones que producen enzi-
Los niveles de andrgenos en el
en la pubertad tambin completa el mas especficas que catalizan cambios
feto masculino permanecen desarrollo como masculino. estructurales y funcionales permanentes.
altos al menos por dos semanas Las hormonas sexuales tienen so- Si estas hormonas no actan en di-
despus del desarrollo de los bre el cerebro dos tipos de efectos: or- cho perodo crtico, su administracin
genitales, lo que sugiere que ganizacional y activante. en estadios posteriores no tiene el mis-
ste puede ser el perodo para la El efecto de la exposicin temprana mo efecto.
diferenciacin sexual del a las hormonas sexuales es un efecto or- La hormona folculo-estimulante
cerebro. ganizacional, que va a determinar la fun- (FSH) es secretada por la hipfisis en
cin cerebral en una forma permanente, los fetos de ambos sexos en la dcima
y que es mediado por la testosterona. semana de gestacin, y su concentra-
Se estima que el perodo crtico del cin aumenta considerablemente en el
desarrollo en el cerebro del feto huma- feto femenino entre las semanas 12 y
no se da en la gestacin temprana e in- 20 de la gestacin. La FSH influye en
termedia, en el tiempo que correlacio- el desarrollo de los ovarios fetales.
na estrechamente con el efecto Durante el desarrollo, y tambin
hormonal que induce la diferenciacin durante la vida adulta, los receptores a
de las gnadas. Los niveles de andrge- los esteroides gonadales se expresan en
nos en el feto masculino permanecen diferentes reas de la corteza cerebral
altos al menos por dos semanas des- que regulan la cognicin y el afecto.
pus del desarrollo de los genitales, lo Los receptores cerebrales mejor estu-
que sugiere que ste puede ser el pero- diados son los receptores a los estrge-
do para la diferenciacin sexual del cere- nos, que regulan la funcin neuronal
bro. El hipotlamo fetal capta la testos- en varias formas importantes, esencial-
terona marcada radiactivamente entre mente previenen la muerte celular y
las semanas 14 y 18 de gestacin. promueven el crecimiento de las cone-

12
Diferencias de gnero y cerebro

xiones celulares, y por lo tanto aumen- rebro. Los estrgenos tienen una fun-
tan la comunicacin neuronal. cin protectora contra los efectos acu-
Sin duda las diferencias en la con- mulativos de la edad, esta funcin pro-
centracin de testosterona y de estr- tectora perdura ms en el cerebro
genos en la vida intrauterina, condicio- masculino que en el femenino. Las mu-
nan las diferencias en el cerebro de jeres son ms propensas a desarrollar en-
hombres y mujeres. El cerebro mascu- fermedad de Alzheimer, an ajustando
lino se desarrolla un poco ms lenta- la prevalencia a las diferencias de expec-
mente que el femenino. Al nacimiento tativa de vida entre hombres y mujeres.
el cerebro masculino es menos maduro En los seres humanos el dimorfis- En la vida adulta la disminucin
y menos simtrico. mo sexual se manifiesta en los siguien- cclica de estrgenos, a la que
Los efectos hormonales no estn li- tes niveles: estn sujetas las mujeres, puede
mitados a las conductas reproductivas y I. Genital. De acuerdo a la apariencia volverlas ms vulnerables a
sexuales, por ejemplo en ratones ma- de los genitales internos y externos. factores estresantes externos. Lo
chos y en ratones hembras, se extienden II. Dimorfismo cerebral. que podra manifestarse como
a todas las conductas en que difieren a. Caviness examin los cerebros de ansiedad o depresin.
machos y hembras: agresin, tendencia 15 hombres y 15 mujeres, de eda-
a los juegos toscos como brincar o bus- des entre 7 y 11 aos, comparndo-
car contacto apoyando el cuerpo. En los los con los cerebros de 20 adultos
hombres y las mujeres el efecto hormo- jvenes: 10 hombres y 10 mujeres.
nal tambin influye, como se ver ms En los cerebros de los menores en-
adelante, en la resolucin de problemas. contr que el cerebro de las nias
Activante. Hay una influencia di- representa 93% del cerebro de los
recta de las hormonas circulantes sobre nios. El 70% de esta diferencia se
el cerebro. Esta influencia aparece debi a un mayor volumen del ce-
cuando el nivel hormonal aumenta. rebelo en los nios que en las nias.
Un ejemplo son los cambios fsicos y Reiss estudi la imagen volumtri-
psicolgicos que experimentan las mu- ca del cerebro de 85 nios entre 5 y 17
jeres antes de la menstruacin, en el aos; encontr que los nios tenan un
embarazo, en el postparto, y en la me- volumen cerebral 10% mayor que el
nopausia, coincidiendo con los cam- de las nias, la diferencia est determi-
bios en los niveles de hormonas feme- nada por un mayor volumen de mate-
ninas, estrgenos y progesterona. ria gris en los nios que en las nias.
Una de las funciones naturales de Utilizando RMN, Giedd estudi a
los estrgenos es su habilidad para 104 nios sanos, de entre 4 y 18 aos
neutralizar los efectos de los glucocor- de edad. Encontr que el volumen ce-
ticoides que se liberan como respuesta rebral en el sexo masculino es 9% ma-
a factores estresantes. yor que en el sexo femenino.
En la vida adulta la disminucin Taairach y Tournoux, mediante im-
cclica de estrgenos, a la que estn su- genes cerebrales comparadas a un atlas es-
jetas las mujeres, puede volverlas ms tereotxico, evaluaron la diferencia en el
vulnerables a factores estresantes exter- volumen cerebral global y regional, usando
nos. Lo que podra manifestarse como anlisis de covarianza tomando al peso cor-
ansiedad o depresin. poral como covariable. Encontraron que el
Durante la menopausia, la secrecin mayor volumen cerebral en hombres que
de hormonas gonadales disminuye y en mujeres es global, y no depende en par-
prcticamente cesa. Por el contrario en ticular de ninguno de los cuatro lbulos.
el hombre, los testculos continan pro- Witelson realiz un estudio post-
duciendo testosterona, la cual es parcial- mortem de nueve sujetos que fallecieron
mente convertida en estradiol en el ce- por cncer. Las mujeres tuvieron un cere-

13
PAC PSIQUIATRA-3 LIBRO 1

bro de menor tamao que los hombres; d. El ncleo preptico en el hipotla-


pero a pesar de ello el cerebro de las mu- mo anterior. Esta regin es mayor
jeres tiene ms neuronas, 11% de ellas se en ratas machos que en ratas hem-
encuentran en dos capas de la corteza ce- bras, estas diferencias son similares
rebral, que estn relacionadas con la en los humanos, y se deben a la pre-
comprensin del lenguaje y el reconoci- sencia de andrgenos en la etapa pre
miento de melodas y tonos de voz. y postnatal.
b. Las regiones neocorticales. Se han e. El ncleo intersticial del hipotlamo
identificado diferencias entre hombres anterior, de acuerdo al reporte de Le-
y mujeres en estudios postmortem. Vay en 1991, es de mayor tamao en
El mayor tamao del cuerpo El cuerpo calloso total y el esple- los hombres que en las mujeres. Y
calloso y del esplenium favorece nium tienen un tamao mayor en las tiene un tamao mayor en hombres
una mejor comunicacin mujeres que en los hombres (cuadro 1). heterosexuales que en homosexuales.
interhemisfrica en las mujeres. Gorski y Allen examinaron 146 ce- El rea del cerebro que organiza la
rebros de cadveres, encontraron que conducta reproductora, femenina o
en las mujeres la parte posterior del masculina, es el hipotlamo, que se co-
cuerpo calloso es 23 veces ms grande necta con la hipfisis y determina la se-
que en los hombres. crecin tnica (masculina), o cclica
El mayor tamao del cuerpo calloso (femenina) de las gonadotrofinas.
y del esplenium favorece una mejor co- f. Se ha propuesto que los andrge-
municacin interhemisfrica en las mu- nos aumentan la potencia funcio-
jeres, y la menor comunicacin interhe- nal del hemisferio derecho.
misfrica en los hombres condiciona Las diferencias de gnero en la late-
asimetra en funciones como el lenguaje. ralizacin del cerebro fueron descritas
c. El ncleo supraquiasmtico en el desde hace ms de 100 aos en estu-
hipotlamo, es mayor en hombres dios postmortem. Se identific que el
que en mujeres. La funcin de este cerebro de las mujeres muestra una
ncleo es la liberacin de las gona- mayor simetra con respecto al peso del
dotrofinas. hemisferio izquierdo y el derecho.

Cuadro 1. Dimorfismo cerebral

REGIN CEREBRAL HOMBRES MUJERES

LBULO TEMPORAL
Regin de la corteza cerebral En hombres cognitivamente sanos, En esta regin las mujeres tienen
encargada de la audicin, la memoria, la delgada regin del lbulo temporal que ms neuronas que los hombres.
el sentido del s mismo y el sentido se encuentra detrs del ojo, tiene 10% Esta regin se encarga de la
del tiempo menos neuronas que en las mujeres comprensin del lenguaje, de las
melodas y del tono de voz
CUERPO CALLOSO
Conjunto de neuronas que constituyen En el cerebro de los hombres el cuerpo calloso La parte posterior del cuerpo calloso es
el puente principal, que comunica el tiene un volumen menor que en las mujeres. mayor en mujeres que en hombres.
cerebro del lado derecho con el del Lo que sugiere que en los hombres la Esto explica porque el cerebro de las
lado izquierdo comunicacin interhemisfrica es menor mujeres utiliza ambos hemisferios en
tareas de lenguaje
COMISURA ANTERIOR
Conjunto de neuronas que conecta En los hombres la comisura es menor En las mujeres la comisura es de mayor
los dos hemisferios. Es ms pequeo que en las mujeres tamao que en los hombres.
y aparece ms temprano a lo largo Esta puede ser otra razn de que en
de la evolucin, que el cuerpo calloso ellas ambos hemisferios trabajen
en colaboracin, en el lenguaje
y en el control de las emociones

14
Diferencias de gnero y cerebro

Bear propone que los niveles circu- estrgenos aumentan el recambio de se-
lantes de testosterona en tero influen- rotonina. En todo el cerebro los estrge-
cian el desarrollo del cerebro: a) pro- nos aumentan la destruccin de la enzi-
moviendo el crecimiento del ma monoaminooxidasa, y de esta
hemisferio derecho, o b) retrasando el manera aumenta la disponibilidad en el
desarrollo del hemisferio izquierdo. cerebro de serotonina y catecolaminas
Diamond encontr que la corteza incluida la noradrenalina que se relacio- La mayora de las diferencias
derecha es ms gruesa que la izquierda na con la respuesta autonmica al es- se han identificado en el
en ratas machos y no en ratas hembras. trs. hipotlamo, como ya se
Stewart propone una influencia hor- Los estrgenos tienen efecto sobre mencion el rea del cerebro
monal temprana, que condiciona esta la dopamina, modifican los receptores que determina el impulso
asimetra. Los andrgenos parecen su- pre y postsinpticos. y la conducta sexuales.
primir el crecimiento de la corteza del Los estrgenos inducen a la enzima
hemisferio izquierdo. En fetos huma- acetilcolintransferasa, lo que tiene un
nos, La Coste, encontr que la corteza efecto benfico sobre la memoria.
del hemisferio derecho es ms gruesa Los estrgenos tienen una accin po-
que la corteza del hemisferio izquierdo tenciadora sobre los aminocidos excita-
en los fetos del sexo masculino. torios como el glutamato y el aspartato.
La asimetra cerebral en los seres Las neuronas que responden al es-
humanos est presente a las 20 sema- tmulo de la progesterona, usan el ci-
nas del embarazo y totalmente consoli- do gamma aminobutrico (GABA),
dada en la semana 36. que es un neurotransmisor inhibitorio,
En resumen podramos decir que la y producen un efecto ansioltico.
mayora de las diferencias se han iden- Debido a su efecto predominante-
tificado en el hipotlamo, como ya se mente excitatorio, los estrgenos tie-
mencion el rea del cerebro que deter- nen un efecto proconvulsivante, en in-
mina el impulso y la conducta sexuales. dividuos susceptibles disminuyen el
Tambin se han encontrado diferencias umbral convulsivo y aumentan la mag-
en la amgdala y en la corteza cerebral. nitud de la actividad convulsiva. Por
III. Neurobioqumico. otro lado la progesterona, aumenta el
Los esteroides gonadales tienen umbral convulsivo, es decir tiene un
efectos irreversibles cuando actan de efecto opuesto a los estrgenos. En resumen podemos decir, que
manera organizacional en perodos cr- En resumen podemos decir, que la la influencia global de la
ticos del desarrollo, y reversibles cuan- influencia global de la progesterona so- progesterona sobre los
do actan de manera activante depen- bre los neurotransmisores, disminuye la neurotransmisores, disminuye la
diendo del nivel srico, por ejemplo ansiedad, pero probablemente aumen- ansiedad, pero probablemente
antes de la menstruacin. ta la depresin. A diferencia de la corre- aumenta la depresin.
En las mujeres la mayor severidad lacin positiva entre el nivel circulante
de los sntomas afectivos (depresivos y de los estrgenos con el aumento en el
ansiosos) est asociada a los perodos estado de alerta, la menor distractibili-
en que las hormonas ovricas, sobre to- dad y la disminucin en los tiempos de
do los estrgenos, se encuentran en ni- reaccin, en animales de laboratorio.
veles bajos, por ejemplo en la fase pre- Ambas hormonas mantienen un ba-
menstrual, en el perodo postparto y lance u homeostasis, que cuando se alte-
en la menopausia. ra, contribuye, en personas vulnerables,
Los estrgenos ejercen efectos ml- al desarrollo de ansiedad o depresin.
tiples en sitios especficos sobre distintos Adems de los efectos sobre los
sistemas de neurotransmisin. En la neurotransmisores, los estrgenos tam-
amgdala, estructura involucrada en la bin aumentan el flujo sanguneo cere-
respuesta al estrs medioambiental, los bral, lo cual acelera el metabolismo y

15
PAC PSIQUIATRA-3 LIBRO 1

Cuadro 2. Tareas que realizan mejor los hombres gan. En ciertos tipos de aprendizaje, en
la resolucin de problemas, en la secre-
1. Tareas espaciales
a) Rotacin mental de un objeto cin de neurotransmisores, en los rit-
b) Imaginar un objeto en tres dimensiones mos circadianos, en la conducta sexual
2. Hacer coincidir los agujeros al doblar una hoja y reproductiva.
3. Dar en el blanco
Las mayores diferencias entre los se-
a) Interceptar proyectiles
4. Encontrar una forma simple oculta dentro de una figura compleja xos en las funciones intelectuales pare-
5. Razonamiento matemtico cen deberse ms bien a los patrones de
6. Habilidad espacial habilidad que al nivel global de inteli-
a) Aprender una ruta en menos ensayos
gencia o coeficiente intelectual (CI).
En promedio, los hombres ejecu-
tan mejor que las mujeres ciertas prue-
aumenta la eficiencia neuronal. Los es- bas, por ejemplo la de rotacin espa-
trgenos estimulan el transporte y el cial, que requiere que el sujeto se
metabolismo de la glucosa. imagine un objeto rotndolo o mani-
Se ha propuesto que los estrgenos pulndolo de alguna manera. Realizan
influyen sobre los genes neurotrficos mejor pruebas de razonamiento mate-
responsables del desarrollo, la migracin, mtico, de navegar a travs de una ru-
el crecimiento y la sobrevivencia neuro- ta, son ms exactos en habilidades mo-
nales; sobre la actividad antioxidante, la toras como "dar en el blanco", guiar o
Los efectos de las hormonas
plasticidad sinptica, la neurodegenera- interceptar proyectiles (cuadro 2).
sexuales sobre la organizacin cin, y la apoptosis secundaria a la falta Las mujeres tienden a ser mejores
cerebral ocurren a las pocas de glucosa o a otras causas txicas. que los hombres respecto a la veloci-
semanas de vida embrionaria, Los estrgenos ayudan a aumentar dad perceptual, es decir la habilidad de
antes de que lo haga las conexiones neuronales. Si se pro- identificar rpidamente reactivos pa-
el medio ambiente. duce una alteracin en un nervio, los reados. Tienen mayor fluidez verbal,
estrgenos reparan, retrasan, o previe- incluida la habilidad para encontrar
nen la atrofia. palabras que empiezan con una letra
IV. Diferencias conductuales. especfica. Son mejores en el clculo
El dimorfismo anatmico y bio- aritmtico y en recordar pistas para se-
qumico condiciona diferencias en la guir una ruta. Son ms rpidas en cier-
conducta de hombres y mujeres. Hay tas tareas de precisin manual, como
diferencias en la forma en que hom- colocar piezas en los espacios apropia-
bres y mujeres se alimentan, juegan (en dos (cuadro 3).
nios juegos ms toscos, entrando en Como ya se mencion los efectos
contacto con el otro), en el nivel de ac- de las hormonas sexuales sobre la orga-
tividad, en la agresividad que desplie- nizacin cerebral ocurren a las pocas
semanas de vida embrionaria, antes de
Cuadro 3. Tareas que realizan mejor las mujeres que lo haga el medio ambiente.
Aunque algunos investigadores in-
1. Velocidad perceptual
a) Identificar pares iguales forman que las diferencias de gnero
2. Recordar objetos en un espacio confinado para resolver problemas no aparecen si-
3. Fluidez verbal no hasta despus de la pubertad, hay
Enlistar:
evidencia de que a los tres aos, los ni-
a) Palabras que empiecen con la misma letra
b) Objetos del mismo color os son mejores que las nias en dar en
4. Precisin en tareas manuales el blanco. En cambio las diferencias en
a) Coordinacin motora fina la habilidad de rotacin espacial se pre-
b) Colocar piezas en los espacios apropiados
sentan antes de la pubertad. En la vida
5. Clculo aritmtico
6. Seguir una ruta ya aprendida a travs de pistas adulta los hombres aprenden una ruta
en menos ensayos y cometen menos

16
Diferencias de gnero y cerebro

errores que la mujeres. Pero cuando ya que la habilidad espacial no aumenta a


est completado el aprendizaje de di- la par que aumenta el nivel de andr-
cha ruta, las mujeres recuerdan mejor genos. Shute determin el nivel de an-
las pistas que los hombres. Esto seala drgenos plasmticos en estudiantes
la posibilidad de que las mujeres tien- hombres y mujeres, y dividi cada gru-
dan a usar las pistas como una estrate- po en nivel alto y nivel bajo de andr-
gia para orientarse en la vida cotidiana, genos. Encontr que las mujeres con
y la estrategia de los hombres est ms niveles altos de andrgenos, tuvieron
relacionada a su habilidad espacial. un mejor desempeo en pruebas de
Tambin hay diferencia en la habilidad habilidad espacial. En los hombres el
para recordar objetos y su localizacin resultado fue inverso, los hombres con
en un espacio confinado, las mujeres niveles bajos de andrgenos tuvieron
son mejores para identificar si un obje- mejor ejecucin en dichas pruebas. Hay estudios que sugieren que
to fue o no desplazado de su lugar, y Gouchie y Kimura correlacionaron la relacin entre los niveles de
son ms exactas para volver a colocar el nivel de testosterona en saliva con el andrgenos y algunas
los objetos en su posicin anterior. razonamiento matemtico. Los hombres habilidades espaciales puede
Las diferencias mencionadas pueden con nivel bajo de testosterona tuvieron ser no lineal.
ser pequeas o muy grandes, los hom- un mejor desempeo que los hombres
bres y las mujeres se superponen enor- con nivel alto. Las mujeres con nivel alto
memente en el desempeo de pruebas de testosterona tuvieron un mejor de-
cognoscitivas, por ello los investigadores sempeo que las mujeres con nivel bajo.
usan variaciones dentro de cada grupo Estos hallazgos sugieren que existe
para graduar apropiadamente las dife- un nivel ptimo de andrgenos: bajo
rencias. Una medida de la variacin de para los hombres y alto para las muje-
las calificaciones dentro de un grupo es res, para lograr un mximo desempeo
la desviacin estndar (DE). Para eva- en la habilidad espacial y en el razona-
luar la magnitud de las diferencias entre miento matemtico (cuadro 4).
hombres y mujeres, se divide la diferen- En las pruebas de velocidad per-
cia entre la DE. El nmero resultante se ceptual no se encontr una relacin
denomina tamao del efecto (size effect). entre el mejor desempeo y el nivel de
Si est por debajo de 0.5 la diferencia se testosterona.
considera pequea. Haier realiz tomografa por emi-
Numricamente las diferencias entre sin de positrones (PET) a 22 estu-
hombres y mujeres son las siguientes: diantes de sexo masculino y a 22 estu-
a) Vocabulario: 0.02. diantes de sexo femenino, mientras
b) Razonamiento no verbal: 0.03. resolvan problemas matemticos. La
c) Razonamiento verbal: 0.17. mitad de los hombres y la mitad de las
El tamao del efecto ms grande se mujeres tuvieron calificaciones mayo-
da en las pruebas de: res a 700 y la otra mitad calificaciones
a) Rotacin espacial: 0.7.
b) Dar en el blanco: 0.75.
Estas son pruebas que los hombres Cuadro 4. Relacin del nivel de andrgenos (testosterona) y habilidades
realizan mejor, y el tamao del efecto
1. Habilidad espacial
refleja que hay ms hombres en el ex-
a) Mayor en hombres con andrgenos bajos
tremo ms alto de la distribucin de la b) Mayor en mujeres con andrgenos altos
calificacin. 2. Matemticas
Hay estudios que sugieren que la a) Mayor en hombres con testosterona baja
b) Mayor en mujeres con testosterona alta
relacin entre los niveles de andrge-
3. Velocidad perceptual
nos y algunas habilidades espaciales a) No hay relacin con el nivel de testosterona
puede ser no lineal. Esto quiere decir

17
PAC PSIQUIATRA-3 LIBRO 1

de 540. Los hombres con calificacio- agresin, y ello podra explicar que
nes ms altas mostraron una actividad tengan una mayor predisposicin a la
ms intensa (mayor flujo cerebral) en violencia que las mujeres.
los lbulos temporales, que los hom- Los hombres tienen menor habili-
bres con calificaciones promedio. Las dad que las mujeres para interpretar las
mujeres con calificaciones ms altas emociones en los rostros de los dems.
no se diferenciaron en la actividad ce- Los hombres tienden a estar ms intere-
rebral, de las mujeres con calificacio- sados por nmeros y detalles, y las mu-
nes promedio. Las mujeres con califi- jeres estn ms interesadas en analizar a
caciones ms altas tuvieron una menor la gente. Los esposos Gur reclutaron a
actividad cerebral (menos flujo cere- un grupo de voluntarios de sexo mascu-
bral) en los lbulos temporales, que lino y femenino, les presentaron rostros
los hombres con calificaciones ms al- de hombres y de mujeres, que mostra-
Esto muestra que en los
tas, an cuando las calificaciones eran ban felicidad y tristeza. Tanto hombres
hombres el mejor desempeo comparables. Esto muestra que en los como mujeres fueron casi infalibles pa-
matemtico est relacionado hombres el mejor desempeo mate- ra reconocer la felicidad. Las mujeres
con una actividad cerebral mtico est relacionado con una acti- reconocieron en 90% la tristeza en ros-
mayor, y en las mujeres el mejor vidad cerebral mayor, y en las mujeres tros de hombres y de mujeres; los hom-
desempeo no implica un el mejor desempeo no implica un es- bres tambin identificaron en 90% la
esfuerzo cerebral mayor que el fuerzo cerebral mayor que el de las tristeza en el rostro de hombres, pero
de las mujeres con desempeo mujeres con desempeo promedio. slo en 70% en el rostro de las mujeres.
promedio.
Gur practic PET con glucosa Evolutivamente esto tiene sentido, ya
marcada radiactivamente, a 37 hom- que los hombres tuvieron que desarro-
bres y a 24 mujeres. Los voluntarios llar hipervigilancia para identificar las
permanecieron 30 minutos antes en re- emociones en el rostro de los otros
poso, y al practicar la PET se les dio la hombres, para poder defenderse.
orden de "poner la mente en blanco". En este estudio la PET mostr que
Los hombres mostraron una mayor ac- el sistema lmbico de los hombres est
tividad en el lbulo temporal, en el sis- ms activo que el de las mujeres, pero
tema lmbico, que es una regin evolu- a pesar de ello los hombres no tienen
tivamente ms primitiva, conocida un mejor desempeo que las mujeres
como "cerebro reptiliano", que contro- para identificar las emociones en el
la la accin ligada a las emociones, es- rostro de la gente.
pecialmente la agresividad y la lucha. George valor la actividad cerebral
Las mujeres mostraron una mayor acti- mediante PET, en 10 hombres y 10
vidad en el giro cingulado, una regin mujeres, mientras se les contaban his-
evolutivamente ms reciente, identifi- torias tristes. En ambos sexos se activ
cada como "cerebro de mamfero", que la parte anterior del sistema lmbico,
controla la expresin compleja de las pero el rea activada, result ser 8 veces
emociones, por ejemplo mostrar el ms grande en las mujeres que en los
enojo por el aspecto y no con golpes. hombres. Esta diferencia podra expli-
"Poner la mente en blanco" o no car porqu las mujeres tienden a depri-
pensar en nada es prcticamente impo- mirse dos veces ms que los hombres.
sible. Cuando se les da esta indicacin Los esposos Shaywitz estudiaron a
los hombres fijan su mente en ftbol y 19 hombres y 19 mujeres sanos con
sexo, mientras que las mujeres lo hacen RMN, mientras realizaban tareas de
en hileras de palabras. lenguaje ortogrfico, semntico e iden-
El estudio muestra que el cerebro tificacin de palabras que rimen. En-
de los hombres est ms activo en re- contraron que al realizar estas tareas en
giones que controlan la accin y la los hombres se activa la regin izquier-

18
Diferencias de gnero y cerebro

da del giro frontal inferior, que est de- edad sobre el cerebro en sujetos nor-
trs de la ceja. En las mujeres la activa- males. En la mayora de estos estudios
cin del giro frontal inferior es ms di- se ha identificado que los hombres
fusa, tanto izquierda como derecha. muestran mayores cambios cerebrales
Los autores especulan que las mujeres con la edad que las mujeres.
son ms elocuentes con el lenguaje que Andreasen encontr diferencias de
los hombres, porque asocian sus senti- gnero en el proceso de envejecimien-
mientos (hemisferio derecho) con su to. El tamao de los ventrculos cere-
razn (hemisferio izquierdo), cuando brales aumenta considerablemente con
usan las palabras. el tiempo; este proceso se inicia en los
Tambin hay diferencias de gnero hombres alrededor de los 40 aos, y en
en la patologa cerebral. las mujeres una dcada despus. La afasia se presenta ms a
Kimura encontr que el dao en el Los esposos Gur y Cowell estudia- menudo en las mujeres cuando
hemisferio derecho ocasiona un menor ron mediante RMN a sujetos sanos, el dao est en la regin anterior
desempeo en la habilidad espacial (ro- dividindolos en dos grupos de edad: del cerebro, y ms en hombres
tacin de un objeto) en ambos sexos. El a) Grupo de adultos jvenes, de 18 a cuando el dao est en la regin
dao en el hemisferio derecho no tiene 40 aos de edad. En el que se in- posterior del cerebro.
un mayor efecto en esta prueba en el cluyeron 53 hombres y 43 mujeres.
hombre que en la mujer, ambos resul- b) Grupo de adultos maduros y de la
tan igualmente afectados. Pero si el da- tercera edad, de 41 a 80 aos de
o se localiza en el hemisferio izquierdo, edad. En el que se incluyeron 17
las mujeres ejecutan peor la prueba de hombres y 17 mujeres.
rotacin de un objeto que los hombres. Encontraron una mayor disminu-
La misma autora encuentra que la cin en los lbulos frontales y tempora-
incidencia de afasia o trastorno del len- les en los hombres. Esto significa que el
guaje es mayor en hombres que en dimorfismo sexual anatmico no es fijo,
mujeres que han sufrido dao en el he- sino que contina evolucionando con la
misferio izquierdo, lo cual puede estar edad. Los autores proponen que las hor-
relacionado al hallazgo de los esposos monas juegan un papel en la promo-
Shaywitz, respecto a que el lenguaje es- cin y/o en la prevencin del proceso de
t ms simtricamente organizado en atrofia cerebral que se da con la edad.
las mujeres que en los hombres. Sholl y Kim sugieren que las regio-
La afasia se presenta ms a menudo nes cerebrales con un mayor nmero de
en las mujeres cuando el dao est en receptores a andrgenos, en las fases
la regin anterior del cerebro, y ms en tempranas de desarrollo, pueden ser
hombres cuando el dao est en la re- ms susceptibles a los efectos de la edad.
gin posterior del cerebro. Murphy estudi mediante RMN a
Debido a que las alteraciones vascu- 35 hombres y 34 mujeres sanos, divi-
lares afectan ms frecuentemente el rea
posterior del cerebro que la anterior,
tanto en hombres como en mujeres, s- Cuadro 5. Dao ocasionado por accidentes vasculares cerebrales
tas tienden a presentar una menor fre-
cuencia de afasia que los hombres. REGIN HOMBRES (%) MUJERES (%)
La apraxia, es decir la dificultad para
Afasias
ejecutar los movimientos de las manos, Dao anterior 29 65
se asocia a dao frontal en el hemisferio Dao posterior 60 12
izquierdo en las mujeres, y a dao poste-
rior en los hombres (cuadro 5). Apraxias
Dao frontal izquierdo 12 71
Hay estudios con imgenes cere- Dao posterior 44 7
brales que investigan el efecto de la

19
PAC PSIQUIATRA-3 LIBRO 1

dindolos en dos grupos de edad: de de las habilidades visoespaciales, que in-


20 a 35 aos y de 60 a 85 aos. Encon- cluyen tareas visoconceptuales, cons-
tr que la prdida de volumen cerebral truccionales, y de memoria.
est relacionada con la edad. Dicha Las mujeres tienen una mayor inci-
prdida de volumen se da en los hom- dencia y prevalencia de demencia de
bres en el volumen cerebral total, pre- Alzheimer (DA) que los hombres. Las
dominantemente en los lbulos fron- mujeres con DA muestran una ejecu-
tales y temporales. Y en las mujeres en cin peor en pruebas de habilidades vi-
el lbulo parietal y en el hipocampo. so-espaciales y de memoria que los
El mismo autor estudi mediante hombres; lo que se relaciona con la ma-
PET con glucosa marcada radiactiva- yor disminucin del metabolismo en el
mente, a 55 hombres y 65 mujeres sa- hipocampo, y la mayor disminucin
nos, entre 21 a 91 aos de edad. En- del tamao del lbulo parietal en las
contr que en relacin a la edad hay mujeres que en los hombres. Ambas re-
una disminucin cerebral del metabo- giones estn involucradas en la memo-
lismo de la glucosa: global, frontal, ria, habilidad visoespacial, y en las alte-
temporal y parietal. Esta disminucin raciones neuropatolgicas de la DA.
del metabolismo es significativamente Los estudios de diferencia de gnero
asimtrica: mayor en el parietal iz- con la edad en el volumen cerebral son
quierdo que en el derecho, mayor en el consistentes en el hallazgo de que con la
frontal derecho que en el izquierdo, y edad, los hombres muestran mayores
en las reas del lenguaje, mayor en el cambios que las mujeres, con una prdida
rea de Broca que en la de Wernicke. mayor en los lbulos frontales y tempora-
Respecto al sexo, los hombres pre- les, y una disminucin mayor en el hemis-
sentan mayor disminucin del meta- ferio izquierdo que en el derecho. En los
bolismo en el hemisferio izquierdo. En hombres hay una mayor incidencia de fac-
las mujeres la disminucin del metabo- tores de riesgo para desarrollar alteraciones
lismo se da por igual en ambos hemis- vasculares, la prdida tisular se relaciona a
ferios o predominantemente en el he- enfermedad cerebral microvascular.
misferio derecho. En las mujeres la Conocer las diferencias de gnero so-
disminucin del metabolismo es signi- bre la morfologa del cerebro, nos permi-
ficativamente mayor que en los hom- te evaluar mejor la interaccin entre las
Las capacidades cognoscitivas bres, en el tlamo y en el hipocampo. diferencias de gnero en el cerebro y la
declinan con la edad avanzada, El metabolismo talmico es significa- presencia de psicopatologa. Casi todos
pero no todas las capacidades tivamente mayor, en las mujeres que en los trastornos psiquitricos tienen dife-
estn igualmente afectadas. los hombres, hasta la edad de 24 aos. El rencias de gnero significativas en: la pre-
metabolismo del hipocampo es significa- valencia, la presentacin de los sntomas,
tivamente menor, en las mujeres que en el curso, la severidad, la comorbilidad, y
los hombres, hasta los 70 aos de edad. a menudo, en la respuesta al tratamiento.
Las capacidades cognoscitivas decli-
nan con la edad avanzada, pero no to- PREVALENCIA
das las capacidades estn igualmente
afectadas. La mayora de las habilidades En la infancia los trastornos del neu-
verbales estn preservadas hasta una rodesarrollo son ms frecuentes en los
edad muy avanzada, mientras que las nios que en las nias, as sucede con:
habilidades visoespaciales y la memoria el retraso mental, los trastornos del
declinan ms rpidamente. Las mujeres aprendizaje, los trastornos en la co-
tienen una menor declinacin en la me- municacin, el autismo, el trastorno
moria verbal que los hombres, mientras por dficit de atencin con hiperacti-
que stos tienen una mejor preservacin vidad y la enuresis.

20
Diferencias de gnero y cerebro

Adems de que estos trastornos son en mujeres, sobre todo en sociedades Despus de la pubertad,
menos frecuentes en las nias, tambin industrializadas en donde hay abun- el trastorno depresivo mayor
los sntomas que los caracterizan se dancia de comida. y el trastorno distmico, son dos
presentan en las nias en forma menos En la tercera edad, la demencia de veces ms comunes en las
severa que en los nios. Alzheimer tiende a ser ms frecuente mujeres que en los hombres.
Despus de la pubertad son signifi- en mujeres que en hombres. De 30 a
cativamente ms comunes en las muje- 50% de las mujeres mayores de 85
res que en los hombres los siguientes aos cursan con un proceso demencial.
trastornos: afectivos (el trastorno de- A lo largo de la vida, 24% de los
presivo mayor y el trastorno distmi- hombres presenta trastornos relaciona-
co), de ansiedad, somatomorfos, diso- dos con el alcohol, presentes nicamen-
ciativos y de la conducta alimentaria. te en 5% de las mujeres. La proporcin
Las mujeres son dos veces ms de trastornos por abuso del alcohol en-
propensas que los hombres para pre- tre mujeres y hombres es de 1/5.
sentar la mayora de los trastornos de Los trastornos por abuso de sustan- En el sexo femenino, los
ansiedad. La prevalencia a un ao de cias, sexuales, y de control de impul- episodios depresivos son ms
la ansiedad generalizada es de 2% en sos, son ms frecuentes en los hombres frecuentes y las mujeres se
los hombres y de 2 a 5% en las muje- que en las mujeres (cuadro 6). quejan de mayor nmero y
res. Un 70% de los pacientes que cur- En el sexo femenino, los episodios mayor severidad de los sntomas.
san con fobia social son mujeres. La depresivos son ms frecuentes y las
agorafobia y el trastorno de pnico se mujeres se quejan de mayor nmero y
presenta en 3% de los hombres y en mayor severidad de los sntomas. En
8% de las mujeres. La frecuencia de los perodos premenstrual, postparto y
crisis o ataques de pnico en las muje-
res aumenta en la fase premenstrual.
Despus de la exposicin a un evento Cuadro 6. Diferencias de gnero en la prevalencia de los trastornos psiquitricos,
traumtico, 19% de los hombres desa- a lo largo de la vida
rrolla trastorno de estrs postraumti-
TRASTORNOS MUJERES HOMBRES RAZN M/H
co, y esto sucede en 31% de las muje-
res. Y 85% de los pacientes en quienes Afectivos
persisten los sntomas de estrs pos- T. depresivo mayor 21.3 12.7 1.68/1
traumtico por ms de un ao, son T. distmico 8.0 4.8 2/1
T. afectivo estacional 6.3 1.0 6/1
mujeres. La proporcin de trastorno T. bipolar I 0.9 0.7 1/1
de estrs postraumtico entre mujeres T. bipolar II 0.5 0.4 1/1
y hombres es de 3/2.
El trastorno bipolar (TBP) de ci- De ansiedad
Fobia social 15.5 11.1 1.5/1
clos rpidos se caracteriza por la pre- Ansiedad generalizada 6.6 3.6 2/1
sencia de cuatro o ms episodios afec- T. de pnico 5.0 2.0 3/1
tivos en un ao. Este subtipo es ms
frecuente en mujeres que en hombres. Conducta alimentaria
Bulimia 1.1 0.1 11/1
Dos tercios de los pacientes con TBP Anorexia 0.5 0.05 10/1
de ciclos rpidos son mujeres. El 70% Esquizofrenia 1.7 1.2 1/1
de los cicladores rpidos y el 46% de Personalidad antisocial 1.2 5.8 1/5
los no rpidos son mujeres.
Abuso de sustancias
Despus de la pubertad, el trastor- Abuso de alcohol
no depresivo mayor y el trastorno dis- sin dependencia 6.4 12.5 1/2
tmico, son dos veces ms comunes en Dependencia de alcohol 8.2 20.1 1/2.5
las mujeres que en los hombres. Abuso de sustancias
sin dependencia 3.5 5.4 1/1.5
El 90% de los trastornos de la con- Dependencia de sustancias 5.9 9.2 1/1.5
ducta alimentaria son diagnosticados

21
PAC PSIQUIATRA-3 LIBRO 1

en la menopausia los sntomas depre- efectos del alcohol o de otras sustancias,


sivos son ms frecuentes y ms severos. y para desarrollar trastornos neurodege-
Cuando las mujeres abusan del al- nerativos como la enfermedad de Alz-
cohol, beben en menor cantidad que heimer o la enfermedad de Parkinson.
los hombres, por lo que en los prime- 3. Trastornos inducidos por eventos
ros aos tienen menos problemas so- estresantes.
ciales, laborales y legales. Por otro lado, Las mujeres son ms susceptibles
en las mujeres, los efectos del alcohol que los hombres a los eventos estresan-
sobre el organismo aparecen a menor tes, en los aos entre la pubertad y la
edad. Lo mismo sucede con las altera- menopausia, cuando los niveles hor-
ciones cognoscitivas, que adems tien- monales fluctan. Y se encuentran
den a ser ms severas en las mujeres protegidas durante el embarazo, cuan-
que en los hombres. do los niveles hormonales se encuen-
Las diferencias de frecuencia en to- tran elevados.
dos estos trastornos pueden ser atribui- 4. Trastornos afectivos.
das a factores genticos, hormonales, Para explicar la diferencia en la fre-
culturales y psicosociales. cuencia de presentacin del trastorno
depresivo mayor entre hombres y mu-
HIPTESIS QUE TRATAN jeres, se han propuesto las siguientes
DE EXPLICAR LA ASOCIACIN hiptesis (cuadro 7):
ENTRE LOS ESTRGENOS
Y LOS TRASTORNOS 1. Utilizacin de los servicios
PSIQUITRICOS de salud

Estas hiptesis han sido propuestas por Las mujeres utilizan ms los servicios
Mary V. Seeman y son las siguientes: de salud, incluidos los de salud mental,
1. Trastornos relacionados al neuro- que los hombres. Es decir, las mujeres
desarrollo. tienden a una mayor conducta de bs-
Las mujeres estn relativamente queda de ayuda que los hombres. Por
protegidas por los efectos organizacio- lo tanto hay mayor proporcin de mu-
nales y activantes de las hormonas, jeres que de hombres deprimidos,
hasta el inicio de la menopausia. cuando se realizan estudios en pobla-
2. Trastornos neurodegenerativos. cin que busca atencin; sin embargo
La declinacin en la produccin de cuando se realizan estudios en pobla-
estrgenos durante la menopausia hace cin general la proporcin de depre-
ms susceptibles a las mujeres a los sin sigue siendo mayor en mujeres,

Cuadro 7. Hiptesis propuestas para explicar la mayor prevalencia de trastorno depresivo mayor (TDM) en mujeres que en hombres

1. Las mujeres buscan ayuda con mayor frecuencia


2. Las mujeres identifican ms fcilmente sus sntomas depresivos
3. En los criterios diagnsticos de TDM hay un sesgo por sexo
4. Las mujeres recuerdan ms sus sntomas depresivos que los hombres
5. El abuso y/o la dependencia al alcohol y la personalidad antisocial son equivalentes masculinos de depresin
6. Genticos. La transmisin gentica predispone a las mujeres al TDM
7. Endocrinos/hormonales. Las hormonas predisponen a las mujeres al TDM
8. Exposicin a eventos adversos. Las mujeres estn ms expuestas a los estresores psicosociales, o les dan mayor importancia
9. Estatus social. Las mujeres viven en condiciones sociales desventajosas en muchas culturas
10. Apoyo social. Las mujeres son ms vulnerables a los efectos de un pobre apoyo social
11. Las mujeres desarrollan un esquema cognoscitivo similar al modelo de desesperanza aprendida
12. Las mujeres estn ms propensas a tener un estilo de respuesta de rumiacin

22
Diferencias de gnero y cerebro

por lo tanto no se trata de un artefacto quejan de menos sntomas (en prome-


debido a la utilizacin de servicios. dio 3), que las mujeres (en promedio 5).
Al cambiar la definicin clnica de de-
2. Sesgo en las respuestas presin, considerando nicamente afec-
dadas en las entrevistas to depresivo de al menos dos semanas
de duracin ms la alteracin ocupacio-
Las mujeres tienden a reconocer ms nal, sin considerar un nmero mnimo
fcilmente sus sentimientos, tanto po- de sntomas, encontraron la misma pro-
sitivos como negativos. De ah deriva porcin de depresin en ambos sexos.
la hiptesis de que cuando se les entre- Ernst y Angst en Zurich encuen-
vista las mujeres aceptan con mayor fa- tran que disminuyendo el nmero de
cilidad la presencia de sntomas depre- sntomas requeridos para hacer el diag-
sivos, porque ellas perciben dichos nstico de depresin, aumenta la pro-
sntomas como femeninos. porcin de hombres con el trastorno.
King y Buchwald realizaron un es- En el Escrutinio Nacional de Co- Diversos investigadores han
tudio en el que hombres y mujeres con- morbilidad (ENC) de Kessler se encon- propuesto que las mujeres con
testaron el Inventario de Depresin de tr que cuando se consideran uno o trastorno depresivo mayor
Beck a un investigador y en forma au- ms sntomas para hacer el diagnstico experimentan ms sntomas
toaplicable. Los autores suponan que de Trastorno Depresivo Mayor depresivos que los hombres.
los hombres reportaran menos snto- (TDM), segn el Manual Diagnstico
mas al investigador que al contestar y Estadstico versin III revisada
ellos mismos el inventario. Sin embargo (DSM-III R), la proporcin de mujeres
no encontraron diferencias entre hom- por hombres es de 1.43, si se conside-
bres y mujeres en la expresin pblica o ran cuatro o ms sntomas es de 1.68, y
privada de los sntomas depresivos. si se requieren ocho sntomas es de 2.5
Clancy y Gove no encontraron di- En sentido opuesto, el Instituto
ferencias de gnero en la necesidad de Nacional de Salud Mental (NIHM) en
aprobacin social o en la propensin a Estados Unidos, encontr que dismi-
responder afirmativamente a las pre- nuir el nmero de sntomas requerido
guntas. Las mujeres no consideran los para hacer el diagnstico de TDM no
sntomas psicolgicos ms socialmente modificaba el hecho de que las mujeres
aceptados que los hombres. presentan mayor frecuencia de TDM
que los hombres.
3. Sesgo en el diagnstico Otra hiptesis propone que la dife-
y en el recuerdo rencia de prevalencia de TDM en
hombres y mujeres se debe a que los
Diversos investigadores han propuesto hombres tienden a olvidar ms los sn-
que las mujeres con trastorno depresi- tomas o los episodios depresivos que
vo mayor experimentan ms sntomas han experimentado en el pasado.
depresivos que los hombres. Por lo tan- De acuerdo al ENC, Kessler espe-
to si se usa el mismo criterio en el n- cula que en comparacin al estudio
mero de sntomas para hombres y para Epidemiological Catchment Area
mujeres, al presentar ellas ms snto- (ECA), en el primero se encontr una
mas, el diagnstico se har con ms diferencia menor entre el nmero de
frecuencia en el sexo femenino. hombres y mujeres deprimidos, por-
Angst y Dobler-Mikola realizaron que se incluy una seccin que inte-
un estudio en pacientes deprimidos, en rrogaba acerca de sntomas depresivos
una comunidad en Suiza. Consideran- a lo largo de la vida, lo que reduca la
do el mismo grado de alteracin ocupa- posibilidad de olvidos en el sexo mas-
cional, encontraron que los hombres se culino. De cualquier manera se encon-

23
PAC PSIQUIATRA-3 LIBRO 1

tr mayor frecuencia de TDM en las En la transmisin gentica de los


mujeres, con una razn mujeres vs trastornos afectivos se han involucrado
hombres de 1.7. a los cromosomas 5, el brazo corto del
cromosoma 11 y el cromosoma X.
4. Equivalentes depresivos Si la transmisin gentica del
TDM estuviera ligada a un locus del
En los hombres se presentan con ma- cromosoma X, y fuera una caractersti-
yor frecuencia el abuso y/o la depen- ca dominante, las mujeres resultaran
dencia del alcohol, y la personalidad ms comnmente afectadas que los
antisocial, y se considera que estos tras- hombres, debido a que tienen dos cro-
tornos son equivalentes depresivos. mosomas X, por lo tanto tendran dos
posibilidades de tener el gen afectado.
5. Genticas Si as fuera la prevalencia por sexo
del TBP tambin debera ser mayor en
Los factores genticos La prevalencia del TBP es de 1%, las mujeres.
involucrados en la gnesis del mientras que del TDM es de 15 a
TDM parecen ser de menor 25%. Los familiares de primer grado 6. Endocrinas-hormonales
magnitud que en el TBP de los pacientes con TBP tienen 8 a 18
veces mayor riesgo de desarrollar TBP, Antes de la pubertad el TDM tiende a
y 2 a 10 veces mayor riesgo de desarro- ser ms frecuente en nios que en ni-
llar TDM que la poblacin general. as. El aumento de la prevalencia del
Los familiares de primer grado de los TDM en el sexo femenino empieza
pacientes con TDM tienen 1.5 a 2.5 entre los 10 y los 14 aos, y as se man-
veces ms riesgo de desarrollar TBP, y tiene a lo largo de la vida. Al llegar a la
2 a 3 veces mayor riesgo de desarrollar tercera edad la diferencia en la preva-
TDM que la poblacin general. La lencia de TDM entre hombres y muje-
concordancia para desarrollar TBP en res se hace menos pronunciada.
gemelos monocigotos es de 33 a 90% La mayor prevalencia de TDM en
y en dicigotos de 5 a 25%. La concor- las mujeres durante los aos de vida re-
dancia para desarrollar TDM en ge- productiva, cuando se presentan cclica-
melos monocigotos es de 50% y en di- mente los cambios en las hormonas hi-
cigotos de 10 a 25% (cuadro 8). pofisiarias y gonadales, hace pensar que
Los factores genticos involucra- en las mujeres el TDM refleja el efecto
dos en la gnesis del TDM parecen de los cambios hormonales sobre el esta-
ser de menor magnitud que en el do de nimo. Parry ha propuesto un
TBP. El modo de herencia permanece modelo de kindling bajo la influencia de
incierto para ambos tipos de trastor- las hormonas reproductivas, en aquellas
nos afectivos. mujeres susceptibles a desarrollar tras-
tornos afectivos. El cambio hormonal
mensual condicionado por el ciclo
Cuadro 8. Herencia en el trastorno bipolar y en el trastorno depresivo mayor
menstrual (CM), que en algunas muje-
RIESGO res se asocia a cambios afectivos, aumen-
EN FAMILIARES ta la probabilidad de que desarrollen un
DE PRIMER GRADO TRASTORNO BIPOLAR T. DEPRESIVO MAYOR TDM independiente del CM.
En probandos con TBP 8 a 18 veces ms 2 a 10 veces ms
que en poblacin general Trastorno disfrico
En probandos con TDM 1.5 a 2.5 veces ms 2 a 3 veces ms premenstrual (TDP)
Concordancia en gemelos:
Monocigotos 33-90% 50%
Dicigotos 5-25% 10-25% Aunque la mayora de las mujeres se
quejan de sntomas displacenteros aso-

24
Diferencias de gnero y cerebro

ciados con el ciclo menstrual, el TDP se en general, con exacerbacin pre-


refiere a un conjunto de sntomas afec- menstrual, durante el embarazo, el
tivos, conductuales, y fsicos, de severi- postparto, o la menopausia, regis-
dad suficiente que interfieren con las ac- trando aquellos tratamientos que
tividades acadmicas, domsticas, hayan sido tiles a la paciente o a
laborales, y en las relaciones interperso- sus familiares. Hay una asociacin bien
nales; y que tienen una relacin tempo- c) Hacer diagnstico diferencial con establecida entre TDM y TDP.
ral con la fase ltea del ciclo menstrual. trastornos que se sabe presentan El 57% de las mujeres con
Los sntomas afectivos ansiosos y depre- exacerbacin de los sntomas en el antecedente de TDM
sivos, son los ms comunes (cuadro 9). perodo premenstrual: alteraciones experimentan exacerbacin
Aproximadamente 2 a 10% de las tiroideas, endometriosis, enferme- premenstrual de los sntomas
mujeres estn severamente afectadas. dad fibroqustica de los senos, epi- depresivos.
Hay una asociacin bien estableci- lepsia, fibromialgia, lupus eritema-
da entre TDM y TDP. El 57% de las toso sistmico, migraa y
mujeres con antecedente de TDM ex- sndrome de colon irritable.
perimentan exacerbacin premens- d) Practicar estudios de laboratorio. Si
trual de los sntomas depresivos. Y la paciente se queja de fatiga o le-
80% de las mujeres con TDP han te- targia, realizar una biometra he-
nido TDM previo, y entre las que ya mtica y un perfil tiroideo para
han tenido hijos, 29% ha presentado descartar anemia o hipotiroidismo.
depresin postparto. En pacientes con sangrado mens-
La evaluacin de las pacientes con trual irregular o con amenorrea se
TDP incluye: debe practicar un perfil tiroideo y
a) Realizar una historia clnica, un determinar el nivel de prolactina
examen fsico completos, y ex- srico. En mujeres mayores de cua-
menes endocrinolgico y gineco- renta aos, que se quejan de san-
lgico. grado menstrual irregular o bo-
b) Recabar antecedentes personales y chornos, se deben determinar los
familiares de trastornos afectivos niveles de FSH y estradiol.

Cuadro 9. Criterios diagnsticos DSM-IV del trastorno disfrico premenstrual (TDP)

A. Durante el ao anterior, en la mayora de los ciclos menstruales (CM), la mayor parte del tiempo en la ltima semana de la fase
ltea, estuvieron presentes cinco o ms de los siguientes sntomas. Los sntomas empezaron a remitir a los pocos das del inicio de
la fase folicular, y desaparecieron en una semana. Al menos uno de los sntomas debera ser 1, 2, 3 4.
1. Afecto marcadamente depresivo, sentimientos de desesperanza, pensamientos autodevaluatorios.
2. Ansiedad marcada, tensin, sensacin de "estar alterada".
3. Labilidad afectiva marcada. Sentimientos sbitos de tristeza, ganas de llorar, o sensibilidad aumentada al rechazo.
4. Enojo o irritabilidad persistente y marcado. Aumento en los conflictos interpersonales.
5. Inters disminuido en las actividades habituales (trabajo, escuela, amigos, pasatiempos).
6. Sensacin subjetiva de dificultad para concentrarse.
7. Letargia, fatiga, prdida de energa marcada.
8. Cambios marcados en el apetito, aumento del mismo, con antojo de alimentos especficos.
9. Insomnio o hipersomnia.
10. Sensacin subjetiva de estar fuera de control.
11. Sntomas fsicos: edema y dolor en los senos, dolor de cabeza, dolor articular o muscular, sensacin de distensin
abdominal, aumento de peso.
B. Los sntomas interfieren marcadamente con las actividades laborales, escolares, sociales, en las relaciones con los dems.
(Evitan las actividades sociales, productividad y eficiencia disminuidas en el trabajo y en la escuela).
C. La alteracin no es meramente una exacerbacin de los sntomas de otro trastorno como el TDM, el trastorno de pnico,
el trastorno distmico, o un trastorno de la personalidad. Pero el TDP s puede asociarse en comorbilidad a estos trastornos.
D. Los criterios A, B y C deben ser confirmados a travs del registro prospectivo, al menos durante dos CM sintomticos
consecutivos. El diagnstico puede hacerse provisionalmente antes de esta confirmacin.

25
PAC PSIQUIATRA-3 LIBRO 1

Cuadro 10. Eficacia de los inhibidores selectivos de recaptura de serotonina, tro de las rutinas diarias le permite a
en el tratamiento del trastorno disfrico premenstrual
la paciente identificar las variaciones
AUTOR/AO FRMACO SUJETOS DOSIS mg/da RESULTADO en los sntomas, pudiendo "predecir-
Sundblad, 1993 Clorimipramina 29 50 + los", de tal manera que puede ajustar
Freeman, 1994 Nefazodona 47 100-600 + sus actividades, intentando reducir
Yonkers, 1996 Paroxetina 14 20-30 +
factores estresantes en la semana pre-
Halbreich. 1997 Sertralina 9 100 +
Steiner, 1997 Fluoxetina 44 20 + menstrual. Se debe recomendar dis-
Young,1998 Sertralina 17 50 + minuir el uso de cafena, sal, alcohol,
Wikander, 1998 Citalopram 69 20 + y nicotina en la semana premenstrual.
Freeman, 1999 Sertralina 31 80 +
Recomendar una buena higiene de
Jermain, 1999 Sertralina 57 100 +
sueo. El ejercicio, la relajacin y la
terapia cognoscitivo-conductual pue-
den mejorar los sntomas.
e) Registrar los hbitos dietticos de Freeman enumera por lo menos
las pacientes, debido a que sustan- nueve estudios doble ciego, publicados
cias como la cafena, la sal, y el al- en la dcada de 1990, que prueban la
cohol pueden empeorar el TDP. eficacia de los ISRS en el tratamiento
f) Indagar si el uso de anticoncepti- del TDP (cuadro 10).
vos hormonales ya sea orales o pa- Adems de los ISRS se han utiliza-
renterales causan cambios en el es- do otros tipos de tratamientos farma-
tado de nimo. colgicos en el TDP:
g) Registrar si la paciente est en tra- a) Estrategias hormonales: estradiol
tamiento con medicamentos que subcutneo o transdrmico 50-100
causan efectos secundarios psiqui- mg; sus efectos secundarios son
tricos como: analgsicos, antihiper- nasea, dolor en los senos, aumen-
tensivos, broncodilatadores, corti- to de peso, no es efectivo por va
costeroides, sedantes. oral. Danazol, andrgeno sintti-
Para explicar la etiologa del TDP co, 200-400 mg diario desde el ini-
se han propuesto alteraciones en los ni- cio de los sntomas al primer da de
veles de estrgenos, progesterona, en la la menstruacin; suprime la ovula-
hormona folculo-estimulante (FSH), cin, puede ocasionar acn, au-
en la hormona luteinizante (LH), en el mento de peso, e hirsutismo.
Adems de los ISRS se han cortisol, en la dihidrotestosterona, en b) Vitaminas y minerales: vitamina B6
utilizado otros tipos de las hormonas tiroideas, en los opioides piridoxina, cofactor en la sntesis de
tratamientos farmacolgicos endgenos y en la serotonina. dopamina y serotonina, mejora la de-
en el TDP. La serotonina es el neurotransmisor presin, la irritabilidad, la fatiga, el
que ms se ha relacionado a la etiopato- edema, el dolor de cabeza, a dosis de
genia del TDP, con base en la mejora 50 a 100 mg/da durante todo el CM.
del trastorno con el uso de antidepresi- El calcio 500 mg dos veces al da, la
vos inhibidores selectivos de la recaptu- vitamina E 400 UI/da, ambos du-
ra de serotonina (ISRS); lo que adems rante todo el CM. El magnesio 360
indirectamente apoya la relacin entre mg una a tres veces al da, de la mi-
el TDP y los trastornos afectivos. tad del ciclo al inicio de la mens-
En el tratamiento del TDP son de truacin. Los tres mejoran la depre-
utilidad ciertas estrategias no farma- sin, el dolor y la fatiga.
colgicas. Se debe educar a la pacien- c) Diurticos, como la hidroclorotia-
te y a su familia acerca de lo que son cida 25 a 50 mg/da disminuyen la
los sntomas premenstruales, para dis- retencin de lquidos, pero pue-
minuir los sentimientos de culpa, ver- den causar hipotensin, mareo e
genza, y desamparo. Llevar un regis- hipocalemia.

26
Diferencias de gnero y cerebro

d) Inhibidores de las prostaglandinas: ro o el segundo das despus del parto,


modulan la respuesta inflamatoria es importante dar informacin a ellas y
y aumentan el umbral al dolor, por sus parejas de la posibilidad que pre-
lo tanto reducen el mismo. El ibu- senten este estado transitorio. No se re-
profen 600 mg dos o tres veces al quiere de un tratamiento mdico ni
da, el cido mefenmico 250 a 500 psiquitrico, es suficiente el apoyo y la
mg dos veces al da y el naproxeno psicoeducacin. Se debe vigilar que la
500 mg una o dos veces al da. Los tristeza no se convierta en depresin
inhibidores de prostaglandinas son postparto. Son factores de riesgo la
efectivos para disminuir el dolor presencia de sntomas depresivos du-
plvico, los clicos abdominales y el rante el embarazo, el antecedente de
dolor de cabeza, pero no modifican depresin o de TDP.
los sntomas afectivos. Pueden usar- Es probable que los sntomas sean Estos tres trastornos pueden
se solo los das sintomticos, pero secundarios a la fluctuacin marcada causar disfuncin en la vida
para que tengan una mayor eficacia de los niveles hormonales despus del familiar y tienen efectos
en el control de los sntomas de do- parto. Los niveles de estrgenos caen negativos en el desarrollo
lor, deben iniciarse 7-10 das antes de 2100 ng/100 mL en las dos sema- del recin nacido.
de la menstruacin. nas previas al parto a 14 ng/100 mL al
e) La bromocriptina, agonista de do- quinto da postparto y los de progeste-
pamina, en dosis de 2.5 mg dos o rona de 16 ng/mL a 3 ng/mL.
tres veces al da, diez das antes del Depresin postparto. De 10 a 15%
inicio de la menstruacin, es til de las mujeres tiene riesgo de desarro-
para disminuir el dolor y el edema llar un episodio depresivo en el pero-
de los senos. Puede causar nasea. do postparto. Casi siempre el cuadro
inicia en las dos semanas posteriores al
Depresin y psicosis postparto parto. Es muy importante detectar el
cuadro por las consecuencias negativas
Los sndromes afectivos postparto se que puede generar en el desarrollo
pueden clasificar en: emocional y cognoscitivo del recin
a) Tristeza postparto, denominada en nacido, y la disfuncin familliar que la
ingls "Maternity Blues". depresin de la madre puede ocasio-
b) Depresin postparto. nar. Las mujeres que sufren depresin Las mujeres que sufren
c) Psicosis postparto. postparto se muestran renuentes a ex- depresin postparto se muestran
Estos tres trastornos pueden causar ternar sus sntomas debido a la expec- renuentes a externar sus
disfuncin en la vida familiar y tienen tativa social de que deben sentirse con- sntomas debido a la expectativa
efectos negativos en el desarrollo del tentas y satisfechas. La familia y los social de que deben sentirse
recin nacido. mdicos atribuyen los sntomas de la contentas y satisfechas.
Tristeza postparto. Es el ms leve y paciente a la reaccin de adaptacin
ms comn de los sndromes afectivos ante el cuidado del recin nacido.
postparto. Se presenta en 80% de las Se han mencionado algunos facto-
mujeres que tienen un hijo. Es una res de riesgo. El antecedente de depre-
reaccin transitoria que inicia dos a sin previa aumenta en 24% el riesgo
cuatro das despus del parto y dura de desarrollar depresin postparto. La
menos de dos semanas. Generalmente depresin en el embarazo aumenta
los sntomas ms intensos se presentan 35% el riesgo. La depresin postparto
al quinto da despus del parto y desa- previa confiere un riesgo de recurren-
parecen rpidamente. Se caracteriza cia de 50%. Los eventos adversos de la
por episodios de llanto, labilidad afec- vida, la falta de apoyo sobre todo de la
tiva, irritabilidad y ansiedad. Dado que pareja, aumentan el riesgo. Los factores
las mujeres son dadas de alta el prime- socioeconmicos, la edad de la madre,

27
PAC PSIQUIATRA-3 LIBRO 1

La etiologa de los sndromes las complicaciones obsttricas y la lac- cosis aguda y la agitacin. Si existe el
afectivos postparto es tancia, aparentemente no influyen en antecedente de trastorno bipolar se de-
desconocida. el desarrollo de la depresin postparto. be iniciar inmediatamente un estabili-
El tratamiento es multifactorial, zador del nimo. Si hay sntomas de-
incluye educacin, disminucin de los presivos, los antidepresivos deben
estresores psicosociales, psicoterapia usarse con precaucin por el riesgo de
individual o de grupo, frmacos anti- precipitar un episodio manaco o com-
depresivos, hospitalizacin y terapia plicar el curso de un trastorno bipolar
electroconvulsiva. con el desarrollo de ciclos rpidos. La
Psicosis postparto. Se presenta en 1 a terapia electroconvulsiva (TEC) es una
2 de cada 1000 nacimientos. Es un alternativa de tratamiento muy impor-
cuadro agudo y severo que se caracteri- tante, sobre todo en pacientes que no
za por: labilidad afectiva, agitacin se- responden a la farmacoterapia, o cuyos
En el tratamiento de los vera, confusin, desorganizacin del sntomas van en aumento.
sndromes postparto primero se pensamiento, alucinaciones e insom- En las mujeres con trastorno bipo-
deben reducir los factores nio. Generalmente el cuadro se desa- lar que tienen mayor riesgo de desarro-
potencialmente estresantes, rrolla dentro de las dos semanas poste- llar psicosis postparto, se debe iniciar
intentar la psicoterapia riores al parto, pero el riesgo se un estabilizador del nimo en el tercer
individual o de grupo, y mantiene hasta tres meses despus del trimestre del embarazo o inmediata-
finalmente considerar la parto. mente despus del parto, para dismi-
farmacoterapia. Entre los factores de riesgo se ha nuir el riesgo de recada.
observado que el antecedente de tras- La etiologa de los sndromes afecti-
torno bipolar manaco depresivo con- vos postparto es desconocida. Se ha in-
fiere un riesgo de 35% de desarrollar volucrado una serie de factores biolgi-
psicosis postparto. Un episodio previo cos, que incluyen los niveles de beta
de psicosis postparto se asocia a 20 a endorfinas, cortisol, estrgenos, hor-
35% de riesgo de recada en un parto monas tiroideas, progesterona, prolac-
subsecuente. Las mujeres con ambos tina y triptfano. En modelos animales,
antecedentes, trastorno bipolar y psi- los estrgenos tienen un efecto antido-
cosis postparto, tienen un riesgo de paminrgico, parecido a los antipsicti-
50% de recurrencia en los partos sub- cos. La deplecin de estrgenos produ-
secuentes. Tienen mayor riesgo las mu- ce una supersensibilidad del receptor a
jeres primparas, aquellas que han re- dopamina y puede predisponer a la psi-
querido de intervencin cesrea, o cosis. Los factores psicolgicos pueden
quienes han sufrido la muerte perina- aumentar la vulnerabilidad para desa-
Las pacientes con psicosis tal del producto. rrollar sntomas depresivos postparto,
postparto pueden incurrir en Las pacientes con psicosis postpar- es el caso de una sensacin disminuida
negligencia para atender al to pueden incurrir en negligencia para de autocontrol, cogniciones maladap-
recin nacido. atender al recin nacido y a los otros tativas, la tensin que genera el emba-
hijos, abuso hacia los hijos, infantici- razo y la llegada del recin nacido.
dio y suicidio. Por ello casi siempre la En el tratamiento de los sndromes
hospitalizacin est indicada. Se debe postparto primero se deben reducir los
descartar etiologa orgnica como ti- factores potencialmente estresantes, in-
roiditis postparto, sndrome de Shee- tentar la psicoterapia individual o de
han, infeccin por el virus de la inmu- grupo, y finalmente considerar la far-
nodeficiencia humana (HIV), estados macoterapia, dado que la mayora de las
de intoxicacin o supresin, trastornos madres desean continuar alimentando
autoinmunes y masa intracraneana. al recin nacido al seno materno. Se de-
El manejo farmacolgico incluye be trabajar en conjunto con el pediatra
antipsicticos para el manejo de la psi- para establecer en el recin nacido un

28
Diferencias de gnero y cerebro

registro de su conducta, de los patrones los cinco a siete aos previos a la me-
de sueo y vigilia y de alimentacin, an- nopausia, es el intervalo entre los CM
tes de iniciar la farmacoterapia. Se debe regulares y el cese total de la funcin
prescribir la dosis mnima necesaria pa- ovrica. El climaterio se refiere a los
ra lograr la remisin de los sntomas. Se aos en que declina la funcin ovrica.
deben preferir los medicamentos de ac- En 1890, Kraepelin acu el tr-
cin corta que los de accin larga. Y se mino de melancola involutiva para re-
puede complementar la alimentacin al ferirse a un sndrome de depresin agi-
seno materno con el uso de frmulas. tada, hipocondriasis, e ideas delirantes
Los medicamentos que estn con- nihilistas, de culpa y vergenza, pre-
traindicados por la Academia America- sente en las mujeres menopasicas. La
na de Pediatra son el litio y los inhibi- menopausia natural ocurre entre los 44
dores de monoamino-oxidasa (IMAOs). y los 55 aos, en promedio a los 52
La concentracin de litio en la leche co- aos. Considerando que el promedio
rresponde al 70% de la concentracin de la expectativa de vida en las mujeres
plasmtica. Los riones del recin naci- en las sociedades industrializadas es de
do son inmaduros, por lo que hay un 81 aos, la menopausia abarca una ter-
riesgo alto de acumulacin de litio, que cera parte de la vida de las mujeres.
puede causar cianosis neonatal, anor- Durante la perimenopausia hay
malidades de la onda T, e hipotona, en cambios hormonales, la funcin ovri-
los nios amamantados por madres tra- ca y la fertilidad declinan. El intervalo
tadas con litio. Los IMAOs pueden in- entre los CM puede acortarse o alargar-
hibir la lactancia y pueden interactuar se, el sangrado menstrual se vuelve ms
con alimentos que contienen tiramina, escaso o ms abundante. Conforme se
con riesgo de causar en el recin nacido acerca la menopausia el sangrado
crisis hipertensivas. menstrual es ms escaso y menos fre-
El bupropin se acumula en la le- cuente, hasta que cesa completamente.
che materna, por lo que se debe evitar En respuesta a la disminucin en la Los sntomas y los signos fsicos
en la lactancia. produccin de estrgenos por parte de propios de la perimenopausia
Las benzodiacepinas (BZD) pue- los ovarios, la hipfisis aumenta la pro- y de la menopausia se deben
den acumularse en el recin nacido por duccin de FSH y LH. Un nivel eleva- a la disminucin en la
la inmadurez del funcionamiento he- do de la FSH srixca en el segundo o produccin de estrgenos.
ptico. Las BZD se pueden utilizar en tercer da despus del inicio de la
forma ocasional en la lactancia, pero se menstruacin, indica que la mujer se
recomienda evitar su uso regular. encuentra en la perimenopausia. En la
Los antidepresivos tricclicos, los menopausia los niveles de estradiol
inhibidores de recaptura de serotonina, permanecen por debajo de 25 pg/mL,
los antipsicticos, los anticonvulsivan- y los niveles de FSH permanecen por
tes se encuentran en la categora de fr- arriba de 40 mUI/mL.
macos cuyo efecto en los recin naci- Los sntomas y los signos fsicos pro-
dos es desconocido, pero deben usarse pios de la perimenopausia y de la meno-
con precaucin; con el cido valproico pausia se deben a la disminucin en la
se debe vigilar el riesgo de hepatotoxi- produccin de estrgenos. Los sntomas
cidad en el recin nacido. vasomotores, sobre todo los bochornos y
los escalofros se presentan en 80% de
Menopausia las mujeres perimenopusicas, y pueden
persistir durante aos hasta el ltimo
La menopausia se define como el pun- CM. Los bochornos son una sensacin
to en el cual cesa permanentemente la de calor extrema que se desarrolla ines-
menstruacin. La perimenoapusia son peradamente en la cara, la parte superior

29
PAC PSIQUIATRA-3 LIBRO 1

del cuerpo, o todo el cuerpo, con una de los mismos, son factores que au-
duracin de 1 a 5 minutos, seguidos de mentan el riesgo de desarrollar snto-
sudor fro. Tambin se puede presentar mas depresivos en la perimenopausia.
dificultad para respirar, mareo, taquicar- La mayora de las mujeres que han
dia. El cuadro clnico se debe diferenciar sido sexualmente activas antes de la me-
del de un ataque de pnico. nopausia, as se mantienen, aunque los
Cuando los sntomas vasomotores cambios fisiolgicos secundarios a la dis-
se presentan por la noche pueden ge- minucin de estrgenos pueden condi-
nerar insomnio, con los consecuentes cionar disminucin de la libido. Los
efectos de la deprivacin de sueo que cambios en el tracto genitourinario pue-
incluyen disminucin en la concentra- den causar sequedad vaginal, infeccio-
cin, fatiga e irritabilidad. nes y dispareunia. La terapia de reem-
La reduccin en la produccin de es- plazo hormonal con estrgenos por va
trgenos puede asociarse con cambios f- oral, vaginal o transdrmica, resulta
sicos como la atrofia del tracto urogeni- efectiva en el tratamiento de los cambios
El tratamiento hormonal de
tal que puede condicionar inflamacin debidos a la atrofia urogenital. Para las
reemplazo se puede administrar urinaria y/o vaginal, infecciones, dispa- mujeres cuyo principal sntoma es la dis-
si la mujer perimenopusica reunia, aumento de la frecuencia urina- minucin de la libido, dosis pequeas
experimenta afecto depresivo ria, urgencia al orinar e incontinencia de testosterona pueden ser tiles.
leve a moderado, irritabilidad, ocasional. La osteoporosis y los trastor- Otros factores que alteran el funcio-
insomnio, disminucin en la nos cardiovasculares son consecuencias a namiento sexual son problemas de sa-
capacidad para concentrarse, largo plazo de la disminucin de los ni- lud crnicos, la depresin, deterioro de
asociados a frecuentes e veles de estrgenos. En mujeres mayores la relacin afectiva con la pareja, funcio-
incmodos bochornos y
de 50 aos la probabilidad de presentar namiento sexual previo no satisfactorio
sudoracin nocturna.
cncer de seno es de 10%, fractura de ca- y disfuncin sexual en la pareja.
dera por osteoporosis es de 15% y enfer- El tratamiento hormonal de reem-
medad coronaria es de 46%. plazo se puede administrar si la mujer
Los estudios longitudinales no han perimenopusica experimenta afecto
identificado un incremento en el riesgo depresivo leve a moderado, irritabili-
de desarrollar TDM en las mujeres me- dad, insomnio, disminucin en la ca-
nopusicas. Sin embargo algunas mujeres pacidad para concentrarse, asociados a
experimentan sintomatologa depresiva, frecuentes e incmodos bochornos y
sobre todo aquellas mujeres que sufren sudoracin nocturna. Este tratamiento
sntomas vasomotores severos, como bo- puede aliviar tanto los sntomas vaso-
chornos o sudoracin nocturna. Cuando motores como los sntomas psicolgi-
los sntomas fsicos se resuelven, tambin cos. La dosis de estrgenos y progeste-
mejoran los sntomas depresivos. rona vara segn las preparaciones; se
Las mujeres que tienen riesgo de pueden administrar estrgenos conju-
desarrollar sntomas depresivos en la gados a dosis de 0.625 mg/da y aceta-
menopausia son aquellas que han pre- to de medroxiprogesterona 2.5 mg/da.
sentado TDP, depresin postparto, Si no hay mejora de los sntomas
cambios afectivos con el uso de anti- depresivos es recomendable el manejo
conceptivos orales, o un TDM previo. psiquitrico combinando el tratamien-
Los problemas de salud crnicos y los to con frmacos antidepresivos y psico-
estresores sociales como estar divorcia- terapia individual o de grupo.
das, viudas, o separadas, tener un bajo
nivel de escolaridad, o estar sometidas a 7. Estatus social
la responsabilidad de cuidar a un fami-
liar de edad avanzada y/o enfermo, si se La hiptesis del estatus social propone
tienen hijos el crecimiento y separacin que las mujeres que juegan un papel

30
Diferencias de gnero y cerebro

femenino tradicional se encuentran so- res estn igualmente expuestos y son


cialmente en desventaja. La mujer tie- igualmente vulnerables a eventos ad-
ne que enfrentar una discriminacin versos de la vida, pero las mujeres estn
econmica, legal y social, que le difi- ms expuestas y son ms vulnerables a
culta alcanzar sus objetivos, y esto con- eventos adversos ocurridos a otros.
tribuye al desarrollo del TDM.
Hay estudios que encuentran que 9. Apoyo social
el matrimonio es un factor protector
contra el TDM en ambos sexos. La falta de apoyo social se ha considera-
Otros estudios encuentran que es un do un factor de riesgo para que las mu-
factor protector contra TDM en el se- jeres desarrollen TDM. Las mujeres sin
xo masculino, y un factor de riesgo un confidente cercano tienen un mayor
para las mujeres casadas que trabajan riesgo de desarrollar depresin que aque-
fuera del hogar, que tienen que cum- llas que s cuentan con dicho apoyo.
plir una doble o triple jornada combi- El apoyo social parece ser particu-
nando el desempeo laboral, con el larmente importante para las mujeres
La falta de apoyo social se ha
papel de esposa, ama de casa y madre. que trabajan fuera del hogar. En esta
considerado un factor de riesgo
Wilhelm y Parker subrayan que las di- poblacin el apoyo social adecuado se para que las mujeres desarrollen
ferencias de gnero en la prevalencia asocia a una menor frecuencia de de- TDM.
del TDM son ms marcadas durante presin y viceversa.
los aos reproductivos, cuando los ro-
les sociales masculino y femenino son 10.Cognoscitivos.
ms divergentes. Desesperanza aprendida
Es importante hacer notar que la
mayor prevalencia de TDM en las mu- El modelo de desesperanza aprendida
jeres es universal, independientemente propone que la depresin es una con-
de la diversidad en la cultura, la polti- secuencia de creer que uno no puede
ca, y la situacin econmica. controlar los eventos externos necesa-
rios para conseguir bienestar.
8. Exposicin y vulnerabilidad Este modelo ha sido ampliado e
a eventos adversos incluye al estilo atributivo asociado a
de la vida depresin, la persona atribuye los
eventos adversos que le acontecen a
Es bien conocido que el TDM es pre- una falla en ella misma, y tiene la ex-
cedido con frecuencia por eventos ad- pectativa de que los eventos negativos
versos de la vida. afectarn su vida en diferentes niveles
Weissman y Klerman consideran y por mucho tiempo. Quienes tienen
que las mujeres pueden ser ms vulnera- este estilo son ms vulnerables a desa-
bles a desarrollar depresin, porque es- rrollar depresin, que quienes tienen
tn expuestas a ms eventos estresantes. un estilo opuesto, en el que atribuyen
Tambin se ha hipotetizado que las los eventos negativos a causas externas,
mujeres pueden ser ms vulnerables a especficas y de corta duracin.
los efectos de los eventos estresantes de Se propone que las mujeres desa-
la vida. Uhlenhuth encuentra el mis- rrollan un esquema cognoscitivo de
mo nivel de factores estresantes, pero la desesperanza aprendida en el proceso
intensidad de los sntomas es aproxi- de internalizar los valores socialmente
madamente 25% mayor en las muje- considerados como femeninos.
res. Turner y Avison estudiaron a 967 Es difcil poder determinar si lo
adultos discapacitados, y encontraron anterior es causa o consecuencia de la
que tanto los hombres como las muje- depresin.

31
PAC PSIQUIATRA-3 LIBRO 1

11.Cognoscitivos. Estilos de propuesto que debido a que las muje-


respuesta res sufren de ms episodios depresivos
tienen mayor probabilidad de recibir
Nolen-Hoeksema propone que los es- tratamiento con frmacos antidepresi-
tilos de respuesta determinan la dura- vos que pueden precipitar los ciclos r-
cin de los episodios depresivos. Los pidos. Adems las alteraciones tiroi-
individuos que desarrollan una res- deas son ms comnes en las mujeres,
puesta de rumiacin a la depresin, stas desarrollan con mayor frecuencia
que pasan la mayor parte del tiempo hipotiroidismo al ser tratadas con litio.
rumiando sus sntomas, prolongan el Villeneuve, Transbol y Cho, han en-
tiempo en que estn deprimidos e in- contrado que de 90 a 100% de los pa-
crementan el impacto de la depresin cientes con TBP que han desarrollado
en su vida cotidiana. En sentido hipotiroidismo secundario al trata-
opuesto, las personas que desarrollan miento con litio, eran mujeres. El hi-
conductas para no estar rumiando to- potiroidismo se ha relacionado con los
do el tiempo los sntomas depresivos, ciclos rpidos, 71% de las mujeres con
acortan la duracin de los mismos. hipotiroidismo son mujeres.
Las mujeres desarrollan un estilo de Wehr y Goodwin proponen que el
respuesta de rumiacin con mayor fre- eje neuroendocrino reproductivo en las
cuencia que los hombres, y esto se aso- mujeres, es el generador de los ciclos r-
cia con perodos ms largos de nimo pidos. En su estudio cuatro de cinco
depresivo, lo que contribuye a la mayor mujeres con TBP de ciclos rpidos pre-
prevalencia de TDM en las mujeres. sentaban CM irregulares, amenorrea,
antecedentes de haber sido tratadas con
DIFERENCIAS DE GNERO estrgenos o progesterona, e inicio del
EN OTROS TRASTORNOS trastorno en el perodo postparto.
PSIQUITRICOS El ciclo reproductivo femenino
afecta el desarrollo de trastorno tiroideo.
Trastorno bipolar Es comn que el bocio aparezca duran-
te la pubertad, el embarazo o la meno-
El TBP de ciclos rpidos que El trastorno bipolar (TBP) tiene una pausia. Las mujeres estn particular-
cursa con cuatro o ms prevalencia de 1% y se presenta en la mente propensas a desarrollar
episodios afectivos en un ao, misma proporcin en hombres que en hipotiroidismo durante el perodo post-
es tres veces ms frecuente en mujeres. Sin embargo hay diferencias parto, y esto puede representar un fen-
mujeres que en hombres. de gnero en cuanto a las manifestacio- meno autoinmune. En los primeros
nes clnicas. Las mujeres con TBP pre- meses del embarazo pueden generarse
sentan ms episodios depresivos que anticuerpos microsomales. Las mujeres
los hombres, y stos tienden a presen- mayores de 40 aos tienen mayor riesgo
tar ms episodios manacos. Los episo- desarrollar disfuncin tiroidea.
dios de mana disfricos (mixtos) son Koukopoulos encuentra que 70%
ms frecuentes en las mujeres. de los pacientes que desarrollaron ciclos
Un hallazgo muy interesante es rpidos en respuesta al tratamiento con
que el TBP de ciclos rpidos que cursa antidepresivos tricclicos eran mujeres,
con cuatro o ms episodios afectivos en de edad madura, o menopusicas.
un ao, es tres veces ms frecuente en Tambin identific que los pacientes
mujeres que en hombres. El 70% de con TBP II (con episodios depresivos y
los cicladores rpidos son mujeres, y episodios hipomanacos) eran ms pro-
slo 46% de los no cicladores rpidos pensos a desarrollar ciclos rpidos, y
son mujeres. No se sabe exactamente not una predominancia del sexo fe-
por qu se observa esta diferencia; se ha menino entre los pacientes con TBP II.

32
Diferencias de gnero y cerebro

Tratamiento del trastorno nes se les aade T3, responden tan rpi-
bipolar damente como los hombres tratados
nicamente con imipramina.
Dado que las mujeres en tratamiento En un estudio Klaiber utiliz 5 a
con litio tienen mayor riesgo de desa- 25 mg de estrgenos conjugados para
rrollar hipotiroidismo es importante tratar sntomas depresivos en mujeres
determinar la funcin tiroidea por lo pre y postmenopusicas, encontrando
menos cada seis meses. mayor mejora en aquellas mujeres que
Las mujeres con TBP pueden pre- recibieron antidepresivo tricclico ms
sentar exacerbacin de los sntomas en estrgenos que las que recibieron tric-
el perodo premenstrual. clico y placebo.
Si se les da tratamiento con carba-
macepina (CBZ), sta induce las enzi- Esquizofrenia
mas hepticas, y disminuye la efectivi-
dad de los anticonceptivos orales, por En estudios en comunidad la prevalen-
lo cual hay riesgo de que la paciente cia de esquizofrenia es de 1%. Prctica-
pueda quedar embarazada. mente es igual en ambos sexos, aunque
Si una mujer con TBP desea emba- tiende a ser un poco mayor en el sexo Respecto al tratamiento de los
razarse y est en tratamiento con esta- masculino que en el femenino, en una episodios depresivos en el TBP o
bilizadores del nimo, hay que advertir proporcin de 1.3 a 2:1. La evolucin, en el TDM, las mujeres son ms
a la paciente y a su pareja sobre los ries- con hospitalizaciones ms frecuentes y sensibles a los tratamientos
hormonales.
gos teratognicos de dichos agentes. prolongadas, puede explicar la aparen-
En el caso del litio si se ingiere en el te mayor prevalencia en los hombres.
primer trimestre del embarazo puede Tambin hay diferencias en la edad
ocasionar malformacin cardaca de de inicio. La esquizofrenia inicia ms
Ebstein con implantacin baja de la temprano en los hombres, entre los 18 y
vlvula tricspide. El cido valproico y los 25 aos de edad, en las mujeres, en-
en menor grado la carbamacepina in- tre los 25 y los 35 aos. Adems a dife-
geridas en el primer trimestre del em- rencia de los hombres, 15% de las mu-
barazo pueden ocasionar defectos de jeres presentan otro pico en la edad de
cierre del tubo neural. inicio a los 40 aos. La esquizofrenia de
Las mujeres con TBP tratadas con inicio tardo, que empieza despus de los
cido valproico pueden presentar alte- 45 aos es ms frecuente en las mujeres.
raciones menstruales, hiperandroge- El que la esquizofrenia inicie en las
nismo, ovarios poliqusticos, sobre to- mujeres casi diez aos despus que en los
do en mujeres de menos de 20 aos. hombres, les permite a stas un mejor fun-
Los antipsicticos pueden generar cionamiento premrbido, con la posibili-
amenorrea, alteraciones menstruales y dad de estudiar, trabajar, casarse y tener hi-
aumento del nivel srico de prolactina. jos, antes del inicio de la enfermedad.
Respecto al tratamiento de los epi- Entre hombres y mujeres hay dife-
sodios depresivos en el TBP o en el rencias en la expresin de los sntomas
TDM, las mujeres son ms sensibles a y en el curso de la enfermedad. Los
los tratamientos hormonales. sntomas positivos, alucinaciones audi-
Prange identific que los hombres tivas e ideas delirantes paranoides, y los
responden a las dosis iniciales de imipra- sntomas afectivos son ms comunes
mina ms rpidamente que las mujeres, en las mujeres. Mientras que los snto-
y que la triyodotironina (T3) aumenta el mas negativos como el aislamiento so-
efecto antidepresivo de la imipramina en cial y el afecto aplanado son ms co-
las mujeres pero no en los hombres. Las munes en los hombres. Las mujeres
mujeres tratadas con imipramina a quie- tienen un curso ms favorable que los

33
PAC PSIQUIATRA-3 LIBRO 1

hombres: menos tiempo de interna- En cuanto al tratamiento, las muje-


miento, menos rehospitalizaciones, res necesitan dosis menores de antipsi-
mayor tiempo sin recadas, y por lo cticos, y responden ms rpido y me-
tanto mejor ajuste social. La esquizo- jor que los hombres. Esta diferencia va
frenia tiende a ser ms benigna, con desapareciendo o revierte con la edad.
sntomas ms leves, en los primeros Se ha propuesto que los estrgenos
aos de la enfermedad; pero al pasar pueden tener un efecto parecido a los
los aos los sntomas psicticos se tor- antipsicticos. Las mujeres hospitaliza-
nan ms severos en las mujeres, y en das con esquizofrenia requieren dosis
los hombres tienden a disminuir. menores de antipsicticos, cuando sus
Dado que la esquizofrenia tiene su niveles de estrgenos estn altos, de lo
origen durante el neurodesarrollo fetal, que se concluye que un aumento en los
las diferencias de gnero en el desarro- niveles de estrgenos aumenta la res-
llo y la organizacin cerebral normal puesta teraputica.
pueden ser relevantes para las diferen- Los antipsicticos aumentan los ni-
cias de gnero descritas en la expresin veles de prolactina, la que a su vez inhi-
de la esquizofrenia. Castle ha identifi- be la liberacin de estrgenos. Los estr-
cado que las anormalidades estructura- genos son necesarios para la maduracin
les cerebrales, como el aumento en el de los folculos ovricos, por lo que un
tamao de los ventrculos y la dismi- estado de hipoestrogenismo puede inhi-
Se ha propuesto la hiptesis de
nucin del volumen del hipocampo, bir la ovulacin y producir amenorrea.
que los estrgenos pueden jugar
un papel protector contra la
parecen ser menos frecuentes en las La amenorrea se presenta si el nivel de
esquizofrenia en las mujeres. mujeres que en los hombres con diag- prolactina es mayor a 60 ng/mL (el nivel
nstico de esquizofrenia. normal de prolactina es de 5 a 25
Se ha propuesto la hiptesis de que los ng/mL). La hiperprolactinemia tambin
estrgenos pueden jugar un papel protec- causa galactorrea. Si el nivel de prolacti-
tor contra la esquizofrenia en las mujeres. na es mayor de 100 ng/mL, se debe
La presencia de estrgenos en la premeno- practicar un estudio de imagen cerebral,
pausia aumenta el umbral de vulnerabili- preferentemente resonancia magntica,
dad a la esquizofrenia, mientras que la dis- para descartar la posibilidad de un tu-
minucin de estrgenos despus de la mor hipofisiario. La hiperprolactinemia
menopausia puede relacionarse a un au- tambin puede deberse a embarazo, lac-
mento del riesgo de desarrollar esquizofre- tancia, prdida de peso, uso de opiceos
nia o de que el cuadro empeore. o de anticonceptivos orales. Para reducir
La evidencia de esta relacin inver- los niveles elevados de prolactina se pue-
sa entre estrgenos y sntomas de esqui- de utilizar un agonista de dopamina co-
zofrenia parte de los estudios realizados mo la bromocriptina a dosis de 2.5 a 7.5
sobre las fluctuaciones de los sntomas mg dos veces al da, esta dosis no parece
a lo largo del CM y de la vida repro- exacerbar los sntomas psicticos.
ductiva de las pacientes esquizofrnicas.
La vulnerabilidad a la recada aumenta Trastornos de ansiedad
en la fase del CM en que los estrgenos
son bajos como en la fase premenstrual, En Estados Unidos los trastornos de
el perodo postparto y la menopausia. ansiedad, son los trastornos psiquitri-
Tanto los sntomas positivos como los cos ms prevalentes, afectan a una de
negativos empeoran cuando el nivel de cada diez personas. Son ms comunes
estrgenos es bajo. En cambio cuando en mujeres que en hombres. El trastor-
los niveles de estrgenos son altos, por no de pnico es dos o tres veces ms fre-
ejemplo en el embarazo, los sntomas cuente en mujeres, y el trastorno de an-
psicticos son leves. siedad generalizada y el trastorno por

34
Diferencias de gnero y cerebro

estrs postraumtico es dos veces ms Blume encontr que la deshidroge-


frecuente en mujeres. La prevalencia nasa alcohlica, la enzima que degrada
del trastorno obsesivo compulsivo al alcohol, es significativamente menos
(TOC) es prcticamente igual para am- activa en las mujeres, por lo cual stas
bos sexos. En diferentes estudios Fla- tienden a intoxicarse ms fcilmente
ment, Lenane y Leonard han encontra- que los hombres, con la misma cantidad
do, que los nios tienen un inicio ms de alcohol por kilogramo de peso. Ade-
temprano prepuberal a los nueve aos, ms las mujeres tienen mayor cantidad
que las nias con un inicio a los once de grasa y menor cantidad de agua que
aos alrededor de la pubertad. Si el los hombres, con lo cual alcanzan mayo-
TOC es de inicio en la infancia se pre- res niveles de alcohol en sangre, pues tie-
sentan ms compulsiones que obsesio- nen menos lquido corporal total en el
nes, los sntomas son ms severos, hay que se diluya el alcohol. En los hombres
mayor comorbilidad con trastorno por se considera que beber fuerte es ingerir
tics, y una mayor posibilidad de tener cuatro copas al da, y en las mujeres be-
un familiar con TOC o con tics. ber fuerte es ingerir una o una y media
Debido a que la prevalencia de los copa al da. Las mujeres desarrollan ms
trastornos de ansiedad es mayor en el rpidamente complicaciones mdicas
sexo femenino, el uso de ansiolticos es por la ingesta de alcohol como lcera
dos veces mayor en las mujeres que en pptica, enfermedad heptica, anemia y
los hombres. atrofa cerebral. Las mujeres alcohlicas
Zerbe ha propuesto que la prepon- tienen mayor riesgo de mortalidad que Debido a que la prevalencia de
derancia de los trastornos de ansiedad los hombres alcohlicos. los trastornos de ansiedad es
en la mujer puede estar relacionada El mismo Blume ha identificado mayor en el sexo femenino, el
con el aumento en las demandas fami- que los factores de riesgo para desarro- uso de ansiolticos es dos veces
mayor en las mujeres que en los
liares y laborales en las mujeres, con los llar alcoholismo en las mujeres son el
hombres.
cambios hormonales en el sexo feme- antecedente de abuso sexual, la comor-
nino, con un menor impulso de la au- bilidad con trastorno antisocial de la
tosuficiencia y de la autoconfianza en personalidad y el antecedente de algn
las mujeres que en los hombres, y con familiar con abuso de sustancias. En
el antecedente ms frecuente en las las mujeres el mayor factor de riesgo
mujeres de abuso fsico y/o sexual. para desarrollar alcoholismo es vivir
Dado que los trastornos tiroideos con una pareja que abusa de sustan-
son ms frecuentes en las mujeres so- cias, lo que a su vez disminuye la posi-
bre todo despus de los cuarenta aos bilidad de un tratamiento exitoso. En
que en los hombres, en las mujeres con las mujeres la depresin tiende a prece-
trastornos de ansiedad se debe descar- der al alcoholismo y en los hombres
tar el diagnstico de hipo o hipertiroi- tiende a ser una consecuencia.
dismo. En mujeres postmenopusicas El abuso de alcohol, alucingenos
con trastornos de ansiedad, al igual y opiceos es mayor en hombres que
que en hombres de cuarenta aos o en mujeres. El abuso de cocana y de
ms, se debe hacer el diagnstico dife- anfetaminas es igual en ambos sexos.
rencial con enfermedad cardiovascular. Sin embargo las mujeres usan ms las
anfetaminas con el propsito de con-
Alcoholismo y abuso trolar el peso. Dado que las mujeres
de sustancias presentan mayor prevalencia de ansie-
dad y depresin, reciben mayor nme-
La prevalencia de alcoholismo es dos ro de prescripciones mdicas de tran-
veces mayor en los hombres que en las quilizantes, y ello genera mayor riesgo
mujeres. de dependencia.

35
PAC PSIQUIATRA-3 LIBRO 1

La sociedad estigmatiza ms a las acompaa de un componente afectivo,


mujeres que abusan de sustancias, que como sucede en el TLP.
a los hombres, por lo cual las mujeres
tienden a negar el abuso. Adems del DESARROLLO DE LA
temor de perder la custodia de sus hi- IDENTIDAD DE GNERO
jos si admiten tener un problema de
abuso de sustancias. En la presentacin clsica de Erickson
sobre el desarrollo de la identidad, la di-
Trastornos de la personalidad visin del trabajo entre los sexos era re-
flejo de su aseveracin de que las aspi-
Aproximadamente 75% de las perso- raciones de los varones eran
nas con trastorno antisocial de la per- principalmente orientadas a su carrera
sonalidad (TAP) son hombres y 75% y hacia compromisos ideolgicos, en
de las personas con trastorno lmite de tanto que las mujeres se centraban al-
la personalidad (TLP) son mujeres. rededor del matrimonio y del cuidado
Aproximadamente 75% de las Para explicar estas diferencias, Sko- de los hijos. Entre los aos 1960 y
personas con trastorno antisocial dol propone que el TAP se relaciona 1970 varios investigadores apoyaron es-
de la personalidad (TAP) son con conductas impulsivas y agresivas, y ta propuesta acerca de las diferencias de
hombres y 75% de las personas el TLP con agresividad impulsiva e gneros. Por ejemplo, los intereses vo-
con trastorno lmite de la inestabilidad afectiva. La proclividad a cacionales eran ms importantes en los
personalidad (TLP) son mujeres. estas emociones y conductas es funda- hombres y los intereses de filiacin eran
mentalmente heredada, es parte del ms importantes en la identidad de las
temperamento. A su vez el medio am- mujeres. Sin embargo, en la ltima d-
biente influye considerablemente en el cada, conforme las mujeres han desa-
desarrollo del estilo de personalidad. rrollado intereses vocacionales, las dife-
Desde la infancia los nios mues- rencias entre gneros estn volvindose
tran ms patrones de conducta exter- similitudes.
nalizada y agresiva, y las nias ms pa- Algunos investigadores creen que el
trones de conducta internalizada e orden de las etapas propuestas por
inhibicin. Esto puede estar relaciona- Ericskon son diferentes para hombres y
do a diferencias genticas y del funcio- mujeres. Se cree que para los hombres
namiento del sistema nervioso central la formacin de la identidad precede
y autnomo. Los hombres tienen ni- a la etapa de intimidad mientras que
veles menores de activacin auton- para las mujeres la intimidad prece-
mica, frontal y serotoninrgica, lo que de a la identidad. Estas ideas son con-
contribuye a una menor socializacin, sistentes con la creencia de que las rela-
una menor inhibicin conductual, un ciones y las ligas emocionales son de
menor desarrollo de las habilidades importancia para las mujeres, mientras
verbales y por lo tanto un mayor po- que la autonoma y el lucro son de ms
La tarea de la exploracin de la tencial antisocial en los hombres que importancia para los hombres. La con-
identidad puede ser ms en las mujeres. ceptualizacin y el desarrollo de la
compleja para las mujeres que
Las nias experimentan ms miedo identidad en las mujeres deben incluir
para los hombres.
que los nios, en la crianza se les re- contenidos interpersonales.
fuerza un patrn de conducta menos La tarea de la exploracin de la
agresivo que a los nios, las nias desa- identidad puede ser ms compleja para
rrollan una conciencia o un "superyo" las mujeres que para los hombres. En
ms fuerte, por lo que experimentan algunas ocasiones puede ser confusa y
ms culpa y depresin, y consecuente- conflictiva especialmente para las mu-
mente cuando desarrollan una altera- jeres que esperan de manera exitosa in-
cin en el control de impulsos, sta se tegrar familia y carrera.

36
Diferencias de gnero y cerebro

Una vez que al nio o a la nia se le del gnero sea en base a la identifica-
asigna el sexo por el obstetra o por el cin. De hecho, muchos nios adop-
pediatra, los padres, los hermanos, y en tan el rol genrico antes de los cinco o
general todos los que lo rodean, empie- seis aos de edad e independientemen-
zan a tratar al nio de manera diferen- te de la presencia del padre.
te. Las expectativas van a ser diferentes La teora del aprendizaje social del
si se trata de un nio o de una nia. En gnero enfatiza que el desarrollo del
general, los padres esperan que los hijos gnero del nio ocurre a travs de la
sean varones. En una investigacin en observacin y la imitacin de la con-
los aos de 1970, 90% de los hombres ducta y a travs de la recompensa y el
y 92% de las mujeres queran que su castigo que se les da a los nios por
primer hijo fuera nio. En un estudio conductas apropiadas o inadecuadas.
ms reciente, los padres todava prefie- A diferencia de la teora de la iden-
ren que el primero sea nio: 75% de los tificacin, el aprendizaje social propone
hombres y 79% de las mujeres. En al- que la atraccin hacia el padre no est
gunos pases un nio varn es preferido involucrada en el desarrollo del gnero.
sobre una nia de tal manera que mu- Los padres a menudo usan recompen-
chas madres abortarn un feto femeni- sas y castigos para ensear a las nias a
no. Por ejemplo, en el sur de Corea ser femeninas y a los nios a ser mascu-
donde la determinacin fetal del sexo es linos. Tambin los coetneos recom-
comn, el nacimiento de los varones pensan y castigan la conducta de gne-
excede a la nias en 14% en contraste ro. Adems con la observacin de los
al promedio mundial de 5%. adultos y de los padres en la casa, en la
Para ayudar a contrarrestar esta escuela, en el vecindario y en la televi-
tendencia, los pases asiticos propusie- sin los nios estn expuestos a una
ron que 1990 fuera el ao de la nia. gran cantidad de modelos que mues-
En aos recientes la idea de que los tran conducta femenina y masculina.
padres son los agentes socializantes cr- Los crticos del aprendizaje social
ticos en el desarrollo del papel genri- arguyen que el desarrollo del gnero no
co ha sido puesta en duda. Los padres se adquiere tan pasivamente. Los padres influyen sobre el
son solamente una de muchas fuentes desarrollo del gnero de los
a travs de las cuales los individuos Influencia de los padres nios por medio del ejemplo.
aprenden los roles genricos. La escue-
la, los medios y otros miembros de la Los padres influyen sobre el desarrollo
familia tambin contribuyen a esto. del gnero de los nios por medio del
Las teoras de la identificacin y ejemplo. Tanto las madres como los
del aprendizaje social intentan explicar padres son importantes psicolgica-
la forma en que los nios adquieren las mente. Las madres tienen la responsa-
aptitudes femenina y masculina. bilidad para la crianza y el cuidado f-
De acuerdo a la teora de la identifi- sico. Los padres ms a menudo tienen
cacin, el nio desarrolla una atraccin interacciones de juego y se les da la res-
sexual al padre del sexo opuesto aproxi- ponsabilidad de que los nios y las ni-
madamente a los cinco o seis aos de as adquieran las conductas sociales.
edad. Renuncia a esta atraccin debido Los padres estn ms involucrados en
a sus sentimientos de ansiedad y subse- la socializacin de sus hijos que en la
cuentemente se identifica con el padre socializacin de las hijas. Los padres
del mismo sexo adoptando inconscien- juegan un papel importante en el desa-
temente las mismas caractersticas. rrollo del gnero pues actan de dife-
Sin embargo, muchos autores del rente forma hacia el hijo y hacia la hi-
desarrollo no creen que el desarrollo ja, ms de lo que lo hacen las madres,

37
PAC PSIQUIATRA-3 LIBRO 1

as contribuyen a la distincin entre los aburran y muchas de ellas simplemente


gneros. Muchos padres animan a sus aprenden a estarse quietas y a sonrer.
hijos y a sus hijas a tomar parte en di- Actualmente hay la tendencia a tomar
ferentes tipos de actividades o de jue- en cuenta las caractersticas de los nios
gos. A las nias se les dan muecas en y las nias en la planeacin de la educa-
la infancia y despus se les asigna el cin. Tambin se promueven los estu-
cuidado de sus hermanitos, se les ense- dios del desarrollo de ambos gneros.
a a que tiendan ms al cuidado de los
dems y a las emociones. Los padres a Desarrollo cognoscitivo
menudo tienen juegos agresivos con del cerebro
sus hijos mas que con sus hijas. En la
De acuerdo a la teora del adolescencia los padres les dan a los ni- De acuerdo a la teora del desarrollo
desarrollo cognoscitivo del os ms libertad que a las nias. cognoscitivo del gnero, los nios reco-
gnero, los nios reconocen su Los padres proporcionan la prime- nocen su gnero despus de que han
gnero despus de que han ra forma de identificar los roles genri- desarrollado un concepto de gnero.
desarrollado un concepto de cos, pero despus ste es un proceso so- Una vez de que consistentemente se
gnero. cial en el que toman parte tambin los conciben a ellos mismos como hom-
compaeros. Los nios con activida- bres o mujeres los nios a menudo or-
des apropiadas a su sexo tienden a ser ganizan su mundo en base al gnero.
recompensados por esto y aquellos que Desarrollada por el psiclogo Kohl-
no lo hacen tienden a ser criticados o berg, esta teora arguye que el desarro-
se les aisla. Los nios muestran una llo del gnero procede de la manera si-
clara preferencia para estar con gente guiente: una nia se da cuenta de que
del mismo sexo y esta tendencia gene- es una nia, quiere hacer cosas de nias
ralmente es ms intensa durante los y hacer cosas de nias es recompensan-
aos intermedios y finales de la infan- te. Habiendo adquirido la habilidad
cia. El juego ha sido identificado co- para categorizar, los nios entonces se
mo "la escuela del gnero" y se seala dirigen consistentemente a estas cate-
que los nios ensean unos a otros la goras de conductas. La teora de
conducta masculina y se les refuerza. Kohlberg est basada en la teora del
Se ha visto, por ejemplo, que en la desarrollo cognitivo de Piaget. Con-
escuela los estudiantes varones estn forme madura cognitivamente el nio,
involucrados en ms interacciones que madura tambin su comprensin del
las mujeres. Adems, los varones reci- gnero. Aunque los nios pequeos
ben ms atencin de los profesores. A pueden aplicar la etiqueta de nio y ni-
los estudiantes varones se les da ms a correctamente, su concepto de g-
apoyo, se les da ms crtica, se les reco- nero es simple y concreto, los nios
noce ms que a las mujeres. preescolares descansan en caractersti-
A pesar de que las nias maduran cas fsicas como el vestido y el peinado
antes y estn listas para el entrenamien- para decidir las diferencias entre hom-
to verbal y matemtico a edad ms bres y mujeres. Las nias son personas
temprana y tienen ms control de las con cabello largo y los nios nunca uti-
habilidades motoras finas antes que los lizan vestidos, muchos nios preescola-
hombres, la educacin se ha construido res creen que puede haber cambios en
principalmente en base al desarrollo de el gnero cuando se corta el pelo o
los varones. Las decisiones acerca de la cuando se tiene una nueva apariencia,
lectura, de cundo pueden hacer divi- todava no tienen el desarrollo cogniti-
siones o cundo pueden escribir estn vo para concebir el gnero como lo ha-
basadas en el desarrollo de los varones. cen los adultos. De acuerdo a Kohl-
Para algunos esto hace que las nias se berg todo el reforzamiento del mundo

38
Diferencias de gnero y cerebro

no modificar este hecho. En la etapa caracteres indeterminados (en idioma


concreta operacional, la tercera etapa ingls) como masculinos en el cuento
en la teora de Piaget a los 6-7 aos de popular de "los tres osos".
edad los nios adquieren la constancia
genrica: un varn es todava un varn ESTEREOTIPOS GENRICOS
independientemente de que utilice
pantalones o falda o que su cabello sea Los estereotipos son categoras que re-
largo o corto. flejan nuestras impresiones y creencias
Cuando el concepto de la constan- acerca de lo que se piensa que es apro-
cia de gnero est claramente estableci- piado para los hombres y para las mu-
do, los nios estn motivados a volver- jeres. Todos los estereotipos, ya sea que
se competentes como nios o nias. estn basados en el gnero, en la etni-
Consecuentemente encuentran las ac- cidad o en alguna otra forma de agru-
tividades de los nios y de las nias re- pamiento, se refieren a una imagen del
compensantes e imitan las conductas miembro tpico de una clase.
de los modelos del mismo sexo. El mundo es extremadamente
Un esquema es una estructura cog- complejo, cada da nos confrontamos Un esquema es una estructura
nitiva que organiza y gua las percep- con miles de estmulos diferentes, y el cognitiva que organiza y gua
ciones individuales, un esquema de g- uso de estereotipos simplifica esta las percepciones individuales,
nero organiza el mundo en trminos de complejidad. Si simplemente asigna- un esquema de gnero organiza
hombre o mujer. La teora del esque- mos una etiqueta a alguien, tenemos el mundo en trminos de
ma de gnero establece que la atencin que pensar mucho menos acerca de lo hombre o mujer.
de un individuo y su conducta se guan que es individualmente. Muchos este-
por motivaciones internas para confor- reotipos son ambiguos, por ejemplo
mar los estndares genricos sociocul- los estereotipos masculino y femenino.
turales y los estereotipos. La teora del Diferentes conductas pueden apoyar
esquema de gnero sugiere que la im- los estereotipos: el crecimiento de ca-
pronta del gnero ocurre cuando los in- bello en la cara es masculino y jugar
dividuos estn listos para codificar y or- con muecas o utilizar lpiz labial es
ganizar la informacin en lneas que femenino. Los estereotipos pueden
son apropiadas o tpicas para los hom- modificarse en base a cambios cultura-
bres y las mujeres en una sociedad. les. Durante algn tiempo, el pelo lar-
Mientras que la teora del desarro- go era femenino, hoy esta caractersti-
llo cognitivo de Kohlberg arguye que es ca es irrelevante en la identificacin de
un prerrequisito cognitivo particular la lo femenino y lo masculino. Las con-
constancia de gnero, la teora del es- ductas populares reflejan estereotipos,
quema de gnero establece que hay una por ejemplo entre los grupos socioeco-
propensin general a responder y a ca- nmicamente bajos, la conducta de ser
tegorizar la informacin en base a roles "duros" se incluye como parte del este-
genricos culturalmente definidos. reotipo masculino.
El gnero se presenta en el lengua- De acuerdo a diferentes estudios, los De acuerdo a diferentes
je que los nios emplean. El lenguaje estereotipos femeninos y masculinos es- estudios, los estereotipos
es fundamentalmente sexista. Por tn muy extendidos. Se cree que los femeninos y masculinos estn
ejemplo, en espaol se tiende a em- hombres son dominantes, independien- muy extendidos.
plear el masculino para referirse tanto a tes, agresivos, orientados a lograr cosas,
hombres como a mujeres; si bien no en tanto que las mujeres tienen una me-
suena extrao decir doctor y doctora, nor autoestima, tienden a ayudar a los
no es usual decir jueza. En una inves- dems. En investigaciones recientes los
tigacin, las madres y sus nios de uno hombres y las mujeres de los pases ms
a tres, aos, casi el 95% vean todos los altamente desarrollados se perciben a

39
PAC PSIQUIATRA-3 LIBRO 1

ellos mismos ms similares entre ellos ms extrema en su retrato de los sexos,


que los hombres y las mujeres de pases especialmente para las nias adolescen-
menos desarrollados. En los pases ms tes. En un estudio stas se mostraron
desarrollados las mujeres asisten a la interesadas principalmente en hacer ci-
universidad y tienen empleos semejan- tas, ir de compras y en su apariencia; ra-
tes a los hombres. Mientras la igualdad ra vez demostraron un inters en la es-
sexual aumenta, los estereotipos de cuela o en actividades relacionadas con
hombres y mujeres disminuyen. profesiones. Las muchachas atractivas
Los estereotipos de gnero tambin generalmente son descritas como con
tienen cambios durante el desarrollo. "aire en la cabeza" y las inteligentes son
Las creencias en los estereotipos son presentadas como poco atractivas.
mayores durante los aos preescolares. Otro tipo de programas muy estereoti-
Tienen un pico en los primeros aos pados son los videos. Las mujeres tien-
de la escuela primaria, y disminuyen den a estar vestidas provocativamente y
en los ltimos aos de la misma. En actuar agresivamente; por ejemplo, en
un estudio reciente se observ que los una escena una mujer empuja a un
estereotipos disminuyen con la edad, hombre al campo y lo abraza y lo besa.
lo que se relacionaba con la adquisi- La programacin de estos videos ha si-
cin de habilidades cognitivas. do descrita por algunos muchachos co-
De especial inters es la forma Los nios encuentran roles femeni- mo un mundo de sueos lleno de mu-
en que las mujeres se muestran nos y masculinos en sus interacciones jeres bellas, hermosas donde hay ms
en la televisin. diarias, sin embargo tambin los me- mujeres que hombres y stas buscan a
dios de comunicacin les proporcionan los hombres y no solamente los buscan
mensajes. De especial inters es la for- sino hasta los asaltan y siempre quieren
ma en que las mujeres se muestran en tener actividad sexual.
la televisin. En los aos de 1970 era Si la televisin puede comunicar
aparente que los personajes femeninos mensajes sexistas e influye en la con-
eran menos competentes que los mas- ducta de los adolescentes, tambin los
culinos, 70% de los protagonistas eran mensajes no estereotipados podran re-
hombres y generalmente se les mostra- ducir la conducta sexista.
Los estereotipos de gnero ba trabajando y a las mujeres se les Los estereotipos de gnero tambin
tambin aparecen en los medios mostraba como amas de casa y en pape- aparecen en los medios impresos. En los
impresos. les romnticos. Los hombres aparecan anuncios de revistas, las mujeres mues-
en trabajos de mayor estatus. En los tran a menudo productos de belleza,
aos de1980, en la televisin se modi- productos de limpieza y productos del
fic un poco la forma en la que eran re- hogar, mientras que los hombres se
presentados los hombres y las mujeres, muestran en los anuncios para carros,
consecuentemente muchos problemas bebidas y viajes. Tanto en los programas
ahora se enfocan a otros tipos de fami- de televisin, como en los anuncios,
lias: como familia con divorciados, se- ahora se muestran imgenes de mujeres
gundos matrimonios, cohabitacin y ms competentes de lo que fueron en el
mujeres con estatus elevados. Aun as pasado, pero los anunciadores an no le
se contina observando que en general dan el mismo estatus que a los hombres.
las imgenes masculinas se muestran El problema de sesgo en cuanto al
como ms competentes que las femeni- gnero en los libros para nios recibi
nas. En una investigacin las adoles- atencin por primera vez a principios de
centes indicaron que las ocupaciones 1970. En 1972 un reporte de textos de
en la televisin estn ms estereotipadas 134 escuelas indic que las nias y las
que en la vida diaria. La televisin di- mujeres eran menos visibles que los ni-
rigida a los adolescentes puede ser la os y los hombres. En el mundo real,

40
Diferencias de gnero y cerebro

menos de 50% de la poblacin son ciente recordar por ejemplo, la Ceni-


hombres, pero los personajes en los li- cienta, dedicada a las labores del hogar
bros para nios eran ms de 70%. No y la Bella Durmiente representada con
solamente las mujeres estaban subrepre- su rueca, Blanca Nieves, atendiendo las
sentadas en los libros sino que tambin labores del hogar de siete hombres
estaban presentadas errneamente. Se (enanos) mineros. Todas ellas "redimi-
presentaban pocas mujeres haciendo das" al casarse con un prncipe!
trabajo fuera de la casa, las que se mos- Recientemente, "la Sirenita", "la
traban ejercan ocupaciones tradicional- Bella" y "Muln" representan un tipo
mente familiares. Las mujeres y los de mujeres un poco diferentes, aunque
hombres eran mostrados con diferentes como en el pasado, para que el final sea
personalidades y realizando diferentes feliz, debe haber una historia amorosa.
tareas. En los libros para nios, los
hombres eran descritos, como inteligen- NEURASTENIA
tes, industriosos, valientes y adquiran
habilidades, fama y fortuna. En contras- En la segunda mitad del siglo 19, junto
te, las mujeres eran pasivas, dependien- con la aparicin de publicaciones peri-
tes, amables, cocinaban y limpiaban. dicas, las mquinas de vapor, el telgrafo,
Esta representacin estereotipada de las los avances en la ciencia, se increment la
mujeres no es nicamente de los libros actividad de las mujeres, cambiando ra-
en ingls. En un anlisis de textos utili- dicalmente la condicin social de stas.
zados en Puerto Rico y en programas Se presentaron nuevas oportunida-
bilinges, las nias generalmente son des para ellas, de educacin y trabajo.
mostradas como ms pasivas, dbiles y Considerar enfermizas a las muje-
dependientes y los nios como valien- res fue una caracterstica prominente
tes, creativos, y persistentes. de fines del siglo 19. Los mdicos aso-
Los libros para nios se estn vol- ciaron explcitamente la epidemia de
viendo menos estereotipados de lo que trastornos nerviosos como la anorexia,
eran hace una dcada o dos. En un es- histeria y neurastenia a la ambicin de
tudio reciente, en 150 libros para nios fin de siglo de las mujeres. Aunque pa-
se analiz el contenido de rol genrico, reciera que la anorexia nerviosa es una
se encontr que la frecuencia con que enfermedad nueva, fue descrita y reci-
se mostraba a las nias y a los nios en bi este nombre por William Gull a fi-
las historias tenda a ser similar, lo que nes del siglo 19 (Bruch, 1979).
no ocurra hace 50 aos. Los roles re- Estas mujeres fueron vistas como el
presentados por las nias y los nios en extremo de la abnegacin y el sacrificio.
los libros han cambiado. Las nias Por siglos, la histeria ha sido el pro-
ahora muestran otro tipo de actividades totipo de enfermedades de las mujeres.
pero siguen siendo demostradas como A fines del siglo 19 el trmino histeria
pasivas y dependientes, los nios oca- era casi intercambiable con femenino.
sionalmente son mostrados como pasi- El repertorio de sntomas y el rpi-
vos y dependientes pero las actividades do paso de uno a otro se compar con
que se muestran son semejantes a las de la labilidad y capricho tradicionalmen-
hace 50 aos. Actualmente, con es- te asociados a la labilidad femenina.
fuerzo, los padres, y los profesores pue- Weir Mitchell llam a la histeria el
den encontrar libros interesantes en los "limbo nosolgico de las enfermedades
que las nias son los modelos. femeninas sin nombre".
Las pelculas de dibujos animados, Cuando las histricas enfermaban,
tradicionalmente haban presentado ya no tenan que representar el papel
los roles ms estereotipados. Es sufi- de hijas o esposas sacrificadas, en lugar

41
PAC PSIQUIATRA-3 LIBRO 1

de esto demandaban servicio y aten- cativos en el papel de la mujer en la


cin de los dems. Los mdicos vean a nueva sociedad industrial moderna. El
estas pacientes como repulsivas, intra- aumento en la educacin de las muje-
tables y manipuladoras. res y el inicio de la aceptacin de la
La neurastenia, otro de los trastor- mujer en varias profesiones, result en
nos de fines del siglo 19 fue una forma lo que se ha llamado "la cuestin de la
mas prestigiada de enfermedad, a pesar mujer", el problema de definir el papel
de que comparta muchos sntomas de la mujer en una sociedad transfor-
con la histeria. Originalmente, la neu- mada por la Revolucin Industrial.
rastenia fue un trastorno descrito como Antes de la industrializacin, las muje-
"nerviosismo americano" por el neur- res haban ocupado claramente papeles
logo George Miller Beard quien obser- definidos y de gran valor en la casa y en
v la relacin entre la organizacin so- el cuidado a la salud, pero con el adve-
cial moderna y la enfermedad nerviosa. nimiento de las fbricas y el mercado
Originalmente, la neurastenia El "nerviosismo americano", fre- capitalista estos roles tradicionales ya
fue un trastorno descrito como cuente entre la gente educada y los in- no fueron valiosos.
"nerviosismo americano" por el telectuales fue visto como una forma Aunque se abrieron ms oportuni-
neurlogo George Miller Beard aceptable de enfermedad en hombres. dades para las mujeres todava hubo
quien observ la relacin entre En este perodo se identific la mascu- considerable conflicto social sobre el pa-
la organizacin social moderna y
linidad con el dinero y la propiedad. El pel de la mujer, muchas mujeres de ese
la enfermedad nerviosa.
mismo Beard sufri de esta enferme- tiempo sintieron que este conflicto jug
dad. Sin embargo, la mayora de los una parte importante en el desarrollo de
pacientes con neurastenia fueron mu- la neurastenia. Margaret Cleaves atri-
jeres, a menudo educadas, urbanas y buy la enfermedad no solamente a la
de clase media. Beard pensaba que las sobrecarga de trabajo, sino ms especfi-
mujeres tenan mayor riesgo que los camente a las ambiciones de la mujer en
hombres al tratar de seguir una carrera una sociedad que era incapaz de llenar
puesto que estaban acostumbradas a las aspiraciones de la nueva mujer.
usar su cerebro "poco y en materias tri- Charlotte Perkins Gilman, que ha-
viales". Para los psiquiatras ingleses, ba sido tratada sin xito, identific su
quienes empezaron a utilizar tambin propia enfermedad como el resultado
el trmino neurastenia, sta no era una de la falta de satisfaccin con su papel
enfermedad nueva, sino un nuevo como esposa y como madre notando
nombre para lo que ellos haban llama- que ella era muy sintomtica en la pre-
do "debilidad nerviosa". El tratamien- sencia de su esposo y su hijo pero sus
to estndar fue la cura de reposo. sntomas se desvanecan cuando estaba
El aumento en la neurastenia Los historiadores han estudiado la lejos de ellos. Estas observaciones fue-
ocurri en el contexto de importancia del gnero en la construc- ron desechadas por Weir Mitchell como
cambios significativos en el cin social de diagnsticos como el de un autoengao y sugiri que viviera una
papel de la mujer en la nueva la neurastenia. Las mujeres sufren este vida tan domstica como fuera posible y
sociedad industrial moderna. desorden con mayor frecuencia y la ra- le prohibi que siguiera escribiendo. Su
zn de esto fue motivo de considerable relacin conflictiva con Mitchell sirvi
especulacin. Los hombres jugaron un como inspiracin para una historia cor-
rol importante en hacer el diagnstico ta que es la crnica de una mujer que
respetable y darle credibilidad, la rela- pasa de un cansancio, de estar exhausta,
cin del gnero con la neurastenia es nerviosa, a una depresin psictica co-
interesante y tiene paralelos con el sn- mo resultado de un tratamiento basado
drome de fatiga crnica. en esta cura de descanso.
El aumento en la neurastenia ocu- Se ha enfocado un inters conside-
rri en el contexto de cambios signifi- rable en la relacin entre los rganos re-

42
Diferencias de gnero y cerebro

productivos y el sistema nervioso y esto chell tuvieron experiencia personal con


fue guiado por el modelo terico domi- la neurastenia y ellos alentaron la res-
nante de esa poca, la irritacin refleja. petabilidad de este diagnstico. Beard
De acuerdo a Beard, la doctrina de la deca que por lo menos 10% de sus pa-
excitacin refleja era el efecto de la acti- cientes eran mdicos varones cuyo po-
vidad del sistema nervioso en los rga- der de observacin superior lo ayud al
nos reproductivos y esta teora fue am- mayor conocimiento de este trastorno.
pliamente aplicada y se volvi parte del Sus historias se refieren a varones
debate acerca de la educacin de las socialmente prominente, lderes en los
mujeres. Observ que pareca casi im- negocios, abogados y miembros del
posible para ninguna mujer sufrir neu- clero; as, la neurastenia se volvi una
rastenia general sin desarrollar ms tar- enfermedad ideal, que era tambin una
de o ms temprano algn problema del marca de xito, esto fue importante
tero o de los ovarios y propuso el uso porque a los pacientes les permiti es-
de tratamientos locales para prevenir capar del diagnstico peyorativo de hi-
daos posteriores. Numerosos mdicos pocondriasis e histeria.
arguyeron que el estudio causaba pobre Hay diferencias importantes entre
La neurastenia fue vista como
funcionamiento de los rganos repro- la histeria, la anorexia y la neurastenia, aceptable y aun se consideraba
ductivos y podra comprometer la ferti- pero a fines del siglo 19, estos tres trmi- una forma respetable de
lidad, tambin se deca que la mujer nos sirvieron para identificar a la misma enfermedad para los hombres.
que estudiaba estaba perdida. mujer infeliz, a las "tres caras de Eva"
Es interesante notar que el concep- como lo ha referido Showalter (1985).
to de irritacin refleja est regresando En la actualidad, el diagnstico de
bajo el disfraz del siglo XX, como el in- sndrome de fatiga crnica es ms fre-
ters en la relacin de la endometriosis cuente entre las mujeres y hay muchos
y el sndrome premenstrual con el sn- paralelos con la dinmica de gnero
drome de fatiga crnica. descrita para la neurastenia. Nuestra
Los hombres que sufran de neuras- propia cultura est contendiendo con
tenia proporcionaban a la enfermedad el papel cada vez mayor de las mujeres
credibilidad y respetabilidad. Para Sho- y la falta de adecuacin entre las ambi-
walter, el hecho de que algunos hom- ciones y las posibilidades sociales.
bres padeciesen este trastorno lo hizo La experiencia clnica sugiere que
diferente de la histeria y de la anorexia entre los que sufren sndrome de fatiga En la actualidad, el diagnstico
nerviosa, que tambin ocurra predo- crnica hay un nmero de mujeres y de sndrome de fatiga crnica es
minantemente en mujeres y que sin hombres que se sienten con conflictos ms frecuente entre las mujeres
embargo no suelen ser vistas tan respe- acerca de su vida y con la dificultad de y hay muchos paralelos con la
tables. La neurastenia fue vista como balancear sus carreras con sus obligacio- dinmica de gnero descrita para
la neurastenia.
aceptable y aun se consideraba una for- nes familiares y sus deseos personales.
ma respetable de enfermedad para los
hombres, en una sociedad capitalista en ORIENTACIN SEXUAL
la que se identificaba la masculinidad
con el dinero y la propiedad. Como es- Aunque relacionadas, la conducta se-
ta enfermedad era causada por exceso xual no es idntica a la orientacin se-
de trabajo y excesiva devocin al deber, xual. Se han desarrollado dos lneas de
ambas cualidades deseables en el mer- evidencia biolgica para explicar la
cado, en algunos casos el diagnstico orientacin sexual, una relacionada
era un motivo de honor. con estructuras cerebrales y otra con
Muchos de los mdicos inicial- aspectos genticos. Los andrgenos
mente involucrados en el estudio de la juegan un papel en el desarrollo del di-
neurastenia incluyendo Beard y Mit- morfismo de los ncleos intersticiales

43
PAC PSIQUIATRA-3 LIBRO 1

del rea preptica. Las neuronas en es- ncleo, pero report que el 2 y el 3
te grupo celular son ricas en receptores eran ms grandes en hombres.
para hormonas sexuales, para andrge- Adems de estudiar el intersticial-3
nos y para estrgenos. Aunque las Levay estudi el 1, el 2 y el 4, en 19
hembras y los machos inicialmente tie- homosexuales que haban muerto de
nen el mismo nmero de neuronas en complicaciones de SIDA y 16 hetero-
el rea preptica medial, una cantidad sexuales seis de los cuales tambin mu-
de testosterona secretada por los test- rieron de SIDA. La orientacin sexual
culos del macho alrededor del naci- de los que no murieron de SIDA no se
miento acta para estabilizar la pobla- determin pero asumiendo la distribu-
cin neuronal. En las hembras, la falta cin similar a la poblacin general, no
de testosterona hace que muchas neu- ms de uno o dos sujetos tenan posi-
ronas mueran, lo que le da su estructu- bilidades de ser homosexuales, tam-
ra tpicamente ms pequea. bin incluy seis mujeres cuya orienta-
A fines de los aos de 1970 Roger cin sexual era desconocida. Como
Gorski de la Universidad de California Allen y Gorski, este autor observ que
en Los Angeles descubri un grupo de el 3 era dos veces ms grande en los
clulas en la regin preptica del hipo- hombres que en las mujeres, pero tam-
tlamo de rata, que era ms grande en bin era entre dos y tres veces ms
machos que en hembras. grande en los hombres heterosexuales
El rea preptica ha sido El rea preptica ha sido implicada que en los homosexuales. En algunos
implicada en la regulacin de la en la regulacin de la conducta sexual homosexuales el grupo celular estaba
conducta sexual como la como la conducta de apareamiento y casi ausente. Los anlisis estadsticos
conducta de apareamiento y en en algunos mecanismos reguladores indicaron que la probabilidad de atri-
algunos mecanismos hormonales. La diferencia en el tama- buir este resultado al azar era de uno en
reguladores hormonales. o de este ncleo se atribuy a diferen- mil, de tal manera que se sugiere que
cias en las exposiciones tempranas a los este dimorfismo est relacionado a la
andrgenos. Posteriormente Bleier y orientacin sexual y al sexo. En contra
Byne en la Universidad de Wisconsin de la propuesta de que esto puede de-
examinando el hipotlamo de varias berse al SIDA, Levay encontr que los
especies de roedores, encontraron que heterosexuales que murieron de SIDA
este ncleo es solamente una parte del tenan el ncleo 3 sin diferencia de los
dimorfismo sexual que involucra a que murieron de otras causas. Ade-
otros ncleos del hipotlamo. Ms re- ms, los grupos celulares en la misma
cientemente varios investigadores han rea preptica, el intersticial 1, 2 y 4 no
buscado los ncleos sexualmente di- eran ms pequeos en las vctimas de
mrficos en el hipotlamo de huma- SIDA. Si la enfermedad estuviese te-
nos. Laura Allen identific cuatro po- niendo un efecto destructivo inespec-
sibles homlogos de los ncleos fico, estos ncleos tambin estaran re-
prepticos de la rata y los design co- ducidos. Despus de completar este
mo ncleos intersticiales del hipotla- estrudio, Levay obtuvo el hipotlamo
mo anterior numerndolos del uno al de un homosexual que muri de cau-
cuatro. Sin embargo, diferentes labo- sas no relacionadas con el SIDA y en-
ratorios que han medido estos ncleos, contr que el volumen del ncleo 3 de
han obtenido resultados inconsisten- este homosexual era menos de la mitad
tes. Dick Swaab en Amsterdam, en- que el ncleo de los heterosexuales.
contr que el ncleo intersticial del t- Los estudios de Levay han sido in-
lamo anterior-1 era ms grande en terpretados como una evidencia de que
hombres que en mujeres, mientras que los factores biolgicos operan directa-
Allen no encontr diferencias en ese mente para determinar la orientacin

44
Diferencias de gnero y cerebro

sexual. Sin embargo, esto es contro- can que las lesiones disminuyen el im-
vertido ya que estos estudios no se han pulso heterosexual, probablemente no
reproducido, adems Levay estudi a es as. Los monos machos presionaban
hombres homosexuales con SIDA, una palanca para tener acceso a las
aunque incluy a algunos heterosexua- hembras ms a menudo despus de su
les con SIDA, no se corrige el proble- operacin que antes. Desafortunada-
ma metodolgico de que cuando mu- mente estos machos no tuvieron opor-
rieron, todos los hombres con SIDA tunidad de interactuar con otros ma-
tenan niveles disminuidos de testoste- chos de tal manera que esto no nos
rona como resultado de la enfermedad dice nada acerca de los efectos en la
o de los efectos colaterales de los trata- motivacin homosexual.
mientos. Hasta la fecha, Levay sola- Otras estructuras tambin han sido Ms recientemente varios
mente ha estudiado un hombre homo- involucradas en la orientacin sexual. investigadores han buscado
sexual que no muri de SIDA. Es Allen report que la comisura anterior, los ncleos sexualmente
posible que los efectos en el tamao del una estructura que participa en la dimrficos en el hipotlamo
ncleo intersticial-3 que l atribuy a transferencia de informacin de uno a de humanos.
la orientacin sexual estuviesen causa- otro hemisferio es ms grande en muje-
dos por las anormalidades hormonales res que en hombres. Ms recientemen-
asociadas con el SIDA. te concluy que la comisura anterior de
El trabajo de Deborah Commins y los homosexuales est feminizada, es
Paulline Yahr de la Universidad de Ca- decir, es ms grande que en los hom-
lifornia, apoya esta hiptesis ya que bres heterosexuales. Despus de corre-
ellas encontraron que el tamao de una gir para el tamao del cerebro, la comi-
estructura en ciertos animales que es sura anterior en mujeres y en hombres
comparable a la de los ncleos prepti- homosexuales fue similar en tamao.
cos de la rata vara con la cantidad de Sin embargo Demeter, Doty y Rin-
testosterona en el torrente sanguneo. go, de la Universidad de Rochester, en-
Otro problema con la interpreta- contraron lo opuesto, las comisuras an-
cin de los estudios de Levay es que su teriores eran ms grandes en hombres
anlisis es impreciso. Levay sugiri que en mujeres. An si los hallazgos de
que tanto el ncleo intersticial-3 como Allen son correctos, el tamao de la co-
los ncleos prepticos de la rata, estn misura anterior por s misma no dira
situados en una regin del hipotlamo nada acerca de la orientacin sexual de
que se sabe participa en la generacin un individuo. Aunque encontr dife-
de la conducta sexual masculina. Sin rencias estadsticamente significativas
embargo, aunque estos ncleos sean en el tamao promedio de las comisuras
destruidos, la rata contina con una anteriores de hombres homosexuales y
conducta sexual adecuada. hombres heterosexuales, 27 de los 30
Slim Pen en Wisconsin, ha sugeri- homosexuales tenan comisuras dentro Otra aproximacin para el
estudio de la orientacin sexual
do que la regin involucrada en la con- del mismo rango que los 30 hombres
ha sido el de la gentica.
ducta sexual de monos rhesus machos heterosexuales con los que los compar.
se localiza encima del rea que es com- Otra aproximacin para el estudio
parable a la ocupada por el ncleo in- de la orientacin sexual ha sido el de la
tersticial-3 en humanos. Los machos gentica. Bailey de la Universidad de
con lesiones en esa regin montaban a Northwestern y Pillard de la Universi-
las hembras menos frecuentemente de dad de Boston, estudiaron a hermanos
lo que lo hacan antes de ser operados, no gemelos y hermanos adoptados, ade-
pero su frecuencia de masturbacin no ms de gemelos idnticos y fraternos;
cambi. Aunque algunos han pro- sus resultados fueron paradjicos. Por
puesto que estas observaciones signifi- ejemplo, en los gemelos idnticos 52%

45
PAC PSIQUIATRA-3 LIBRO 1

fueron concordantes y 22% de los ge- de hombres homosexuales tambin son


melos fraternos para la orientacin ho- homosexuales. Para las mujeres, aproxi-
mosexual. Esto apoyara la participa- madamente 50% de gemelas idnticas,
cin gentica ya que los gemelos 16% de gemelas fraternas y 13% de mu-
idnticos comparten todo el material jeres lesbianas tambin son lesbianas.
Cuando esto se compara con la
gentico. Sin embargo, los hermanos Cuando esto se compara con la frecuen-
frecuencia de homosexualidad no gemelos tuvieron un 9% de homo- cia de homosexualidad en la poblacin
en la poblacin general, sexualidad y la hiptesis gentica pro- general, es evidente que hay cierto agru-
es evidente que hay cierto pondra que los hermanos no gemelos pamiento familiar en la orientacin se-
agrupamiento familiar deberan de tener una concordancia xual en ambos sexos.
en la orientacin sexual igual a la de los gemelos fraternos por- Bailey en la Universidad de North-
en ambos sexos. que comparten la misma cantidad de western estim que la heredabilidad
material gentico. Adems, estos auto- global de la orientacin sexual, es decir,
res encontraron que la incidencia de ho- la proporcin de la varianza en un ras-
mosexualidad en los hermanos adopta- go que proviene de los genes, es de
dos de los homosexuales era mucho ms 53% para los hombres y 52% para las
alta que la que se encuentra en pobla- mujeres. Hamer encontr que el her-
cin general. En los hermanos fue de mano de un hombre homosexual tena
11% y en la poblacin general se ha es- 14% de probabilidad de ser homose-
timado de 1 a 5%. Este porcentaje es xual comparado con 2% de los hom-
idntico al de los hermanos biolgicos y bres sin hermanos homosexuales; este
el estudio claramente sugiere la contri- estudio utiliz una definicin restrin-
bucin ambiental a la orientacin se- gida de homosexualidad por lo que las
xual. Otros dos estudios de gemelos cifras son menores que en otros estu-
tambin han encontrado una mayor dios. Entre los parientes ms distantes
concordancia en los gemelos idnticos se observ que los tos maternos tenan
que en los fraternos. Sin embargo todos 7% de probabilidades de ser homose-
estos gemelos han sido criados juntos xuales y los hijos de las tas maternas
por lo que no puede descartarse el efec- 8%; los padres, tos paternos y todos
to del ambiente. Es necesario estudiar a los otros primos del lado paterno no
gemelos criados por separado. Quiz el tuvieron ninguna correlacin.
hallazgo principal de estos estudios es Hamer estudi familias con dos
que a pesar de tener los mismos genes, homosexuales, en un estudio de liga-
haber compartido el ambiente prenatal, miento de genes en el cromosoma X y
y tener un ambiente postnatal lo ms si- encontr un gen candidato a influir en
milar posible, en 50% los gemelos idn- la orientacin sexual, en el brazo largo
ticos fueron discordantes para la orien- de este cromosoma.
tacin sexual. Esto indica lo poco que La concordancia para homosexuali-
se sabe sobre los determinantes de la dad en gemelos homocigotos, menor a
orientacin sexual. 100%, indica que el efecto del ambien-
En general, los estudios relacionados te no puede ser desdeado. Identificar
con los aspectos de la herencia de la ho- los factores del ambiente que influyen
mosexualidad en hombres, muestran en la orientacin sexual permitir com-
que 50% de gemelos idnticos, 24% de prender ms claramente la relacin en-
gemelos fraternos y 13 % de hermanos tre lo biolgico y lo sociocultural.

46
BIBLIOGRAFA
1. Allen LS, Gorski RA. Sexual dimorphism of self reports. American Journal of Sociology brain development: ages 4-18. Cereb
the anterior commissure and massa intermedia 1974;80:205-216. Cortex 1996;6:551-560.
of the human brain. Journal of Comparative 17. Commins D, Yahr P. Adult testosterone 30. Gladue BA, Bailey JM. Spatial ability,
Neurology 1991; 312(1):97-104. levels influence the morphology of a sexu- handedness, and human sexual orientation.
2. Allen LS, Hines M, Shryne JE, Gorski ally dimorphic area in the Mongolian ger- Psychoneuroendocrinology 1995; 20(5):
RA. Two sexually dimorphic cell groups in bil brain. Journal of Comparative 487-97,1995.
the human brain. Journal of Neuroscience Neurology 1984;224(1):132-40. 31. Gladue BA, Bailey JM. Aggressiveness,
1989;9(2):497-506. 18. Cowell PE, Turetsky BI, Gur RC, competitiveness, and human sexual orien-
3. Allen LS, Richey MF, Chai YM, Gorski Grossman IR, Shtasel DL, Gur RE. Sex tation. Psychoneuroendocrinology 1995;
RA. Sex differences in the corpus callosum differences in aging of the human frontal 20(5):475-85.
of the living human being. Journal of and temporal lobes. The Journal of 32. Gouchie C, Kimura D. The relationship
Neuroscience 1991;11(4):933-42. Neuroscience 1994;14:4748-4755. between testosterone levels and cognitive
4. Angst J, Dbler-mikola A. Do the diagnos- 19. Davis EC, Shryne JE, Gorski RA. ability patterns. Psychoneuroendocrinology
tic criteria determine the sex ratio in Structural sexual dimorphisms in the 1991;16:323-334.
depression? J Affect Disord 1984;7:189- anteroventral periventricular nucleus of 33. Gur RC, Mozley LH, Mozley PD, Resnick
198. the rat hypothalamus are sensitive to SM, Karp JS, Alavi A, Arnold SE, Gur RE.
5. Bailey JM, Pillard RC, Dawood K, Miller gonadal steroids perinatally, but develop Sex differences in regional cerebral glucose
MB, Farrer LA, Trivedi S, Murphy RL. A peripubertally. Neuroendocrinology metabolism during a resting state. Science
family history study of male sexual orienta- 1996;63(2):142-8. 1995;267:528-531.
tion using three independent samples. 20. De Lacoste MC, Adesanya T, Woodward 34. Hamer DH, Hu S, Magnuson VL, Hu
Behavior Genetics 1999;29(2):79-86. DJ. Measures of gender differences in the N, Pattatucci AM. A linkage between
6. Bear D, Schiff D, Saber J. Quantitative human brain and their relationship to DNA markers on the X chromosome and
analysis of cerebral asimetries. Arch Neurol brain weight. Biological Psychiatry male sexual orientation. Science
1986;43:598-603. 1990;28(11):931-42. 1993;261(5119):321-7.
7. Benbow CP, Stanley JC. Sex differences in 21. De Lacoste MC, Holloway RL, 35. Hampson E. Spatial cognition in humans:
mathematical ability: fact or artifact? Woodward DJ. Sex differences in the fetal possible modulation by androgens and
Science 1980;210(4475):1262-4. human corpus callosum. Human estrogens. Journal of Psychiatry &
8. Benbow CP, Stanley JC. Sex differences in Neurobiology 1986;5(2):93-6. Neuroscience 1995;20(5):397-404.
mathematical reasoning ability: more facts. 22. Demeter S, Ringo JL, Doty RW. 36. Hampson E. Variations in sex-related cog-
Science 1983;222(4627):1029-31. Morphometric analysis of the human cor- nitive abilities across the menstrual cycle.
9. Bleier R, Byne W, Siggelkow I. Cytoarchi- pus callosum and anterior commissure. Brain & Cognition 1990;14(1):26-43.
tectonic sexual dimorphisms of the medial pre- Human Neurobiology 1998;6(4):219-26. 37. Hedges LV, Nowell A. Sex differences in
optic and anterior hypothalamic areas in guinea 23. Diamond M, Dowling G, Johnson R. Mental Test Scores, variability, and num-
pig, rat, hamster, and mouse. Journal of Morphologic cerebral cortical asimmetry in bers of high-scoring individuals. Science
Comparative Neurology 1982;212(2):118-130. male and female rats. Exp Neurol 1995;269:41-45.
10. Blume SB. Gender differences in alcohol- 1981;71:261-268. 38. Hines M, Allen LS, Gorski RA. Sex dif-
related disorder. Harvard Review of 24. Ernst C, Angst J. The Zurich study XII: sex ferences in subregions of the medial nucle-
Psychiatry 1994;2:7-14. difference in depression: evidence from us of the amygdala and the bed nucleus of
11. Bruch H. The Golden Cage. The enigma longitudinal data. Eur Arch Psychiatry Clin the stria terminalis of the rat. Brain
of anorexia nervosa. Random House, New Neurosci 1992;241:22-230. Research 1992;579(2):321-6.
York, 1979. 25. Flament MF, Whitaker A, Rapoport JL. 39. Holloway RL, De Lacoste MC. Sexual
12. Byne W. The biological evidence chal- Obsessive compulsive disorder in adoles- dimorphism in the human corpus callo-
lenged. Scientific American 1994;26-31. cence: an epidemiological study. J Am Acad sum: an extension and replication study.
13. Castle DJ, Murray RM. The neurodevelop- Child Adolesc Psychiatry 1988;27:764-771. Human Neurobiology 1986;5(2):87-91.
mental basis of sex differences in schizo- 26. Freeman EW. Premenstrual syndromes and 40. Kessler RC, McGonagle KA, Swartz M.
phrenia. Psychol Med 1991;21:565-575. antidepressant treatments. Primary Psychiatry Sex and depression in the National
14. Caviness VS, Kennedy DN, Richelme C. 2000;7;51-55. Comorbidity Survey, I:lifetime prevalence,
The human brain age, 7-11 years: a volu- 27. George MS, Ketter TA, Post RM. SPECT chronicity and recurrence. J Affect Disord
metric analysis based on magnetic reso- and PET imaging in mood disorders. J 1993;29:85-96.
nance images. Cereb Cortex 1996;6:726- Clin Psychiatry 1993;64:6-13. 41. Kessler RC, McGonagle KA, Zhao S.
736. 28. George MS, Ketter TA, Parekh PI. Lifetime and 12 month prevalence of
15. Cho JT, Bone S, Dunner DL. The effects Gender differences in regional cerebral DSM-III R psychiatric disorders in the
of lithium treatment on thyroid function blood flow during transient self-induced United States: results from the National
in patients with primary affective disorder. sadness or happiness. Biol Psychiatry Comorbidity Study. Arch Gen Psychiatry
Am J Psychiatry 1979;136:115-116. 1996;40:859-871. 1994a;51:8-19.
16. Clancy K, Gove W. Sex differences in men- 29. Giedd J, Snell J, Lange N. Quantitative 42. Kessler RC, McGonagle KA, Nelson CB.
tal illness: an analysis of response bias in magnetic resonance imaging of human Sex and depression in the National

47
PAC PSIQUIATRA-3 LIBRO 1

Comorbidity Survey, II: cohort effects. J 57. Murphy DGM, Decarli CH, Mcintosh 71. Tannen Deborah, Aries Elizabeth.
Affect Disord 1994b;30:15-26. AR. Daly E. Sex differences in human Conversational style: Do women and men
43. Kimura D. Sex differences in the brain. brain morphometry and metabolism: An in speak different languages? Walsh, Mary
Scientific American 1992;81-87. vivo quantitative magnetic resonance imag- Roth (Ed); et al. (1997). Women, men, &
44. Kimura D. Sex and cognition. ing and positron emission tomography gender: Ongoing debates. (pp. 79-100).
Cambridge, MA, USA: The Mit Press study on the effect of aging. Arch Gen New Haven, CT, USA: Yale University
1999, x, 217. Psychiatry 1996;53:585-594. Press. XXVI, 445 pp.
45. Kimura D. Sex differences in the brain. 58. Nolen-hoeksema S, Morrow J, Fredrickson 72. Transbol I, Christiansen C, Baastrup PC.
(1993). Mind and brain: Readings from BL. Response styles and the duration of Endocrine effects of lithium, I:hypothy-
Scientific American magazine. (pp. 79- depressed mood. J Abnorm Psychol roidism, its prevalence in long-term treated
89). New York, NY, USA: W. H. Freeman 1993;102:20-28. patients. Acta Endocrinol 1978;87:759-767.
& Co, Publishers. VII, 147 pp. 59. Pattatucci AM, Hamer DH. Develop- 73. Turner RJ, Avison WR. Gender and
46. King DA, Buchwald AM. Sex differences in ment and familiality of sexual orientation depression: assessing exposure and vulnera-
subclinical depression: administration of in females. Behavior Genetics 1995; bility to life events in a chronically strained
the Beck Depression Inventory in public 25(5):407-20. population. J Nerv Ment Dis
and private disclosure situations. J Pers Soc 60. Pillard RC, Bailey JM. Human sexual 1989;177:443-455.
Psychol 1982;42:963-969. orientation has a heritable component. 74. Uhlenhuth EH, Paykel, ES. Symptom
47. Klaiber EL, Broverman DM, Vogel W. Human Biology 1998;70(2):347-65. intensity and life events. Arch Gen
Estrogen therapy for severe persistent 61. Prange AJ, Wilson IC, Rabon AM. Psychiatry 1973b;28:473-477.
depressions in women. Arch Gen Enhancement of imipramine antidepres- 75. Villeneuve A, Gautier J, Jus A. The effect of
Psychiatry 1979;36:550-554. sant activity by thyroid hormone. Am J lithium on thyroid in man. Int J Clin
48. Koukopoulos A, Reginaldi P, Laddomada Psychiatry 1969;126:457-469. Pharmacol 1974;9:75-80.
GF. Course of the manic depressive cycle 62. Reiss AL, Abrams MT. Singer HS. Brain 76. Volkow ND, Wang GJ, Fowler JS,
and changes caused by treatments. development, gender and IQ in children: a Hitzemann R, Pappas N, Pascani K, Wong
Pharmacopsychiatry 1980;13:156-167. volumetric imaging study. Brain CH. Gender differences in cerebellar
49. Lenane M, Swedo S, Leonard H. 1996;119:1763-1774. metabolism: Test-Retest Reproducibility.
Psychiatric disorders in first degree relatives 63. Ringo JL, Doty RW, Demeter S. Bi-ver- Am J Psychiatry 1997;154:119-121.
of children and adolescents with obsessive sus monohemispheric performance in split- 77. Weissman MM. The myth of involutional
compulsive disorder. J Am Acad Child brain and partially split-brain macaques. melancholia. JAMA 1979;242:742-744.
Adolesc Psychiatry 1990;29:407-412. Experimental Brain Research 1991; 78. Weissman MM. Klerman GL. Sex differ-
50. Leonard HL, Lenane MC, Swedo SE. Tics 86(1):1-8. ences and the epidemiology of depression.
and Tourettes syndrome: a 2 to 7 year follow- 64. Seeman MV. Are gender differences in psy- Arch Gen Psychiatry 1977;34:98-111.
up study of 54 obsessive-compulsive chil- chopathology due to hormones? Primary 79. Weissman MM. Klerman GL. Gender and
dren. Am J Psychiatry 1992;149:1244-1251. Psychiatry 2000;7:47-50. depression. Trends Neurosci 1985;8:416-420.
51. Levay S, Hamer DH. Evidence for a bio- 65. Shaywitz BA, Shaywitz SE, Pugh KR. Sex 80. Wehr TA, Goodwin FK. Rapid cycling in
logical influence in male homosexuality. differences in the functional organization manic depressives induced by tricyclic anti-
Scientific American 1994;20-25. of the brain for language. Nature depressants. Arch Gen Psychiatry
52. Levay S. A difference in hypothalamic 1995;373:607-609. 1979;36:555-559.
structure between heterosexual and homo- 66. Showalter E. The female malady. Women, 81. Wehr TA, Sack DA. Rosenthal NE. Rapid
sexual. Science 1991;253(5023):1034-7. madness, and english culture, 1803-1980. cycling affective disorder: contributing fac-
53. Levay S, Hamer DH. Evidence for a bio- Pantheon Books, New York, 1985. tors and treatment responses of 51 patients.
logical influence in male homosexuality. 67. Skodol AE. Gender-Specific etiologies for Am J Psychiatry 1988;145:179-184.
Scientific American 1994;270(5):44-9. antisocial and borderline personality disor- 82. Wilhelm K, Parker G. Is sex necessarily a
54. Maccoby EE, Jacklin CN. The psychology ders? In Gender and its effects on psy- risk factor for depression? Psychol Med
of sex differences. Stanford Univ. Press, chopathology, 37-60. American Psychiatric 1989;19:401-413.
Stanford CA. 1974. Press, Washington, 2000. 83. Williams CL, Barnett AM, Meck WH.
55. Mcewen BS, Lieberburg I, Chaptal C, 68. Sart BL, Goy RW. Heterosexual, autosex- Organizational effects of early gonadal
Davis PG, Krey LC, Maclusky NJ, Roy ual and social behavior of adult male rhesus secretions on sexual differentiation in spa-
EJ. Attenuating the defeminization of the monkeys with medial preoptic-anterior tial memory. Behavioral Neuroscience
neonatal rat brain: mechanisms of action of hypothalamic lesions. Brain Research 1990;104(1):84-97.
cyproterone. Hormones & Behavior 1978; 142(1):105-22. 84. Witelson S. Sexual differentiation of the
1979;13(3):269-8. 69. Swaab DF, Fliers E. A sexually dimorphic human temporo-parietal region for functional
56. Meaney MJ, Mcewen BS. Testosterone nucleus in the Human Brain. Science asymmetry: neuroanatomical evidence.
implants into the amygdala during the 1985;228:1112-1115. Psychoneuroendocrinology 1991;16:131-153.
neonatal period masculinize the social play 70. Talairach J, Tournoux P. Co-Planar stereo- 85. Zerbe KF. Anxiety disorder in women.
of juvenile female rats. Brain Research taxi Atlas of the human brain. New York, Bull Menninger Clin 1995;59 (suppl
1986;398(2):324-8. Thieme Medical Publishers, 1988. A):A38-A52.

48
AUTOEVALUACIN FINAL

1 La sensibilidad mxima del sistema nervioso central al efecto Ver respuestas


organizacional temprano de los esteroides gonadales ocurre en la pgina 51
entre las semanas:
a. 4 a 6
b. 8 a 10
c. 10 a 12
d. 14 a 16
e. 20 a 22

2 El cerebro femenino en relacin al masculino:


a. Se desarrolla un poco ms lentamente durante la vida
intrauterina
b. Es menos maduro al nacimiento
c. Es menos simtrico al nacimiento
d. Todas las anteriores
e. Ninguna de las anteriores

3 Son trastornos ms frecuentes en las mujeres, EXCEPTO:


a. Trastorno bipolar de ciclos rpidos
b. Enfermedad de Alzheimer
c. Esquizofrenia
d. Fobia social
e. Trastorno distmico

4 Son psicofrmacos de eleccin para el tratamiento del trastor-


no disfrico premenstrual:
a. Imipramina y desimipramina
b. Sertalina y fluoxetina
c. Alprazolam y clonazepam
d. Carbamacepina y cido valproico
e. Litio

5 Diez a 15% de las mujeres tienen riesgo de resarrollar:


a. Tristeza posparto "maternity blues"
b. Depresin posparto
c. Psicosis posparto
d. Trastorno bipolar posparto
e. Esquizofrenia posparto

49
PAC PSIQUIATRA-3 LIBRO 1

6 Se presenta en uno a dos de cada 1,000 nacimientos y se ca-


racteriza por labilidad afectiva, agitacin grave, confusin,
desorganizacin del pensamiento, alucinaciones e insomnio:
a. Tristeza posparto "maternity blues"
b. Depresin posparto
c. Psicosis posparto
d. Trastorno bipolar posparto
e. Esquizofrenia posparto

7 En relacin con las mujeres, los hombres con esquizofrenia:


a. Inician entre los 25 y los 35 aos
b. Presentan ms sntomas afectivos
c. Presentan menos sntomas negativos
(aislamiento social, aplanamiento afectivo)
d. Tienen menos rehospitalizaciones
e. Duran menos tiempo sin recada

8 En relacin con los hombres, las mujeres usan ms:


a. Alcohol
b. Alucingenos
c. Benzodiacepinas
d. Cocana
e. Opiceos

9 De acuerdo a los estereotipos imperantes en nuestra sociedad:


a. Las mujeres son ms agresivas que los hombres
b. Los hombres tienden a ayudar ms a los dems
c. Las mujeres tienen una mayor autoestima
d. Los hombres estn ms orientados a lograr cosas
e. Las mujeres son ms independientes

10 Durante el desarrollo en la infancia:


a. Los nios experimentan ms miedo que las nias
b. Las nias desarrollan un superyo ms fuerte
c. Los nios experimentan ms culpa
d. Las nias desarrollan un patrn ms agresivo de conducta
e. Las nias prefieren los juegos que implican contacto fsico

50
RESPUESTAS
A LAS AUTOEVALUACIONES

INICIAL FINAL

1. F 1. d
2. V 2. e
3. F 3. c
4. V 4. b
5. V 5. b
6. F 6. c
7. V 7. e
8. V 8. c
9. F 9. d
10. V 10. b

51
Reg. 208M92, 339M95, 318M2001, 157M95

Вам также может понравиться